Special Rules Relating to Funding Relief for Single-Employer Pension Plans under PRA 2010

n2011-03_Final_2011_omb2196.pdf

Notice - Special Rules Relating to Funding Relief for Single-Employer Pension Plans under PRA 2010 and Notice - Funding Relief for Multiemployer Defined Benefit Plans under PRA 2010

Special Rules Relating to Funding Relief for Single-Employer Pension Plans under PRA 2010

OMB: 1545-2196

Document [pdf]
Download: pdf | pdf
Part III. Administrative, Procedural and Miscellaneous

Funding Relief for Single-Employer Pension Plans under PRA 2010

Notice 2011-3

I. PURPOSE
This notice provides guidance on the special rules relating to funding relief for
single-employer defined benefit pension plans (including multiple employer
defined benefit pension plans) under the Preservation of Access to Care for
Medicare Beneficiaries and Pension Relief Act of 2010 (PRA 2010), Pub. L.
No.111-192.
II. BACKGROUND
Section 430 of the Internal Revenue Code (Code) specifies the minimum funding
requirements that apply to single-employer defined benefit pension plans
pursuant to § 412. For purposes of calculating the minimum required
contribution, § 430 generally requires a plan to establish a shortfall amortization
base with respect to a plan year for which the value of a plan’s assets is less
than the amount of the plan’s funding target. Section 430(c)(2) generally
provides for amortization of a shortfall amortization base over 7 years.
Section 201(b)(1) of PRA 2010 adds § 430(c)(2)(D) which permits a plan sponsor
to elect, in lieu of the otherwise applicable amortization schedule, to amortize the
shortfall amortization base established for certain plan years under one of two
alternative amortization schedules: the 2 plus 7-year amortization schedule, or
the 15-year amortization schedule. The 2 plus 7-year amortization schedule is
described in § 430(c)(2)(D)(ii) and the 15-year amortization schedule is described
in § 430(c)(2)(D)(iii). Section 201(b)(2) of PRA 2010 amends § 430 by adding
§ 430(c)(7), which provides for an acceleration of the required installments under
an alternative amortization schedule in the case of certain compensatory
payments, dividends, and stock redemptions.
Under § 430(c)(2)(D)(v), an election to use an alternative amortization schedule
may generally be made only with respect to one or two eligible plan years, and,
under § 430(c)(2)(D)(iv)(II), if the plan sponsor makes the election for two plan
years, the same amortization schedule must be used for both plan years. An
eligible plan year is a plan year that begins in 2008, 2009, 2010, or 2011, but
only if the due date for the minimum required contribution to the plan for such
plan year under § 430(j)(1) occurs on or after June 25, 2010 (the date of
enactment of PRA 2010). Section 430(c)(2)(D)(iv)(III) provides that any such

election may be revoked only with the consent of the Secretary, after consultation
with the Pension Benefit Guaranty Corporation.
Pursuant to § 430(c)(2)(D)(vi), a plan sponsor that makes an election under
§ 430(c)(2)(D) for a plan year is required to provide notice of the election to
participants and beneficiaries of the plan. Under § 430(c)(2)(D)(vi)(II), the plan
sponsor must also inform the Pension Benefit Guaranty Corporation of such
election in such form and manner as the Director of the Pension Benefit
Guaranty Corporation may prescribe.
Sections 104, 105, and 106 of the Pension Protection Act of 2006 (PPA ’06),
Pub. L. No. 109-280, provide that the effective dates for the minimum funding
rules under § 430 and funding-based benefit restrictions under § 436 are delayed
for certain plans. For plans described in section 104 or 105 of PPA ‘06, these
provisions do not generally apply for plan years beginning before January 1,
2017, and January 1, 2014, respectively. For plans described in section 106 of
PPA ‘06, the provisions of §§ 430 and 436 of the Code do not apply for plan
years beginning before January 1, 2011.
Section 202(a) of PRA 2010 amends Title I of PPA ’06 to allow a plan sponsor of
a plan described in section 104 or 105 of PPA ’06 to elect, for any two eligible
plan years (using the same definition as applies under § 430), one of two
alternative amortization schedules with respect to a portion of the plan’s
unfunded new liability. The schedules, set forth in sections 107(b) and 107(c) of
PPA ’06, as amended by PRA 2010, are generally similar to the 2 plus 7-year
amortization schedule and the 15-year amortization schedule.
Section 202(a) of PRA 2010 also provides for the election of one of the
alternative amortization schedules for plans described in section 106 of PPA ’06.
Such plans are subject to the minimum funding rules of § 430 of the Code for
plan years beginning on or after January 1, 2011, and the election to use an
alternative amortization schedule under section 202(a) of PRA 2010 is available
for these plans only for one eligible year beginning in 2008, 2009, or 2010.
Sponsors of these plans may also make an election under section 201(b)(1) of
PRA 2010 to use an alternative amortization schedule to amortize the shortfall
amortization base for a plan year beginning in 2011.
Section 202(b) of PRA 2010 amends section 104 of PPA ‘06 to provide a delayed
effective date for application of the minimum funding requirements of § 430 and
the funding-based benefit restrictions under § 436 to certain plans maintained by
eligible charities. Under this provision, eligible charity plans (certain plans
maintained by employers described in § 501(c)(3)) generally will not be subject to
the rules of §§ 430 and 436 for plan years beginning before January 1, 2017.
However, plan sponsors may elect to have the provisions of §§ 430 and 436
apply for plan years beginning after December 31, 2007, and on or before
December 31, 2008.

2

Section 303(c)(2) of the Employee Retirement Income Security Act of 1974, as
amended (ERISA), is parallel to § 430(c)(2) of the Code, and section 201(a)(1) of
PRA 2010 amends section 303(c)(2) of ERISA in a manner parallel to the
amendments made to § 430(c)(2) of the Code by section 201(b)(1) of PRA 2010.
Section 201(a)(2) of PRA 2010 adds section 303(c)(7) of ERISA, which is parallel
to new § 430(c)(7). Under section 101 of Reorganization Plan No. 4 of 1978 (43
FR 47713), the Secretary of the Treasury has interpretive jurisdiction over the
subject matter of this notice for purposes of ERISA as well as the Code. Thus,
this notice applies for both purposes.
Notice 2010-55 (2010-33 I.R.B. 253) states that the Service expects to issue
future guidance on the special funding rules under PRA 2010 for single-employer
plans. Notice 2010-55 also states that, in the case of a plan year that ends
before the guidance is issued, the plan sponsor will be permitted to elect to use
an alternative amortization schedule under PRA 2010 without regard to whether
the Form 5500 (and Schedule SB) has been filed for that plan year. This notice
constitutes the guidance anticipated in Notice 2010-55.
III. QUESTIONS AND ANSWERS
The questions and answers in this notice relate to the following topics:
G - General rules
I - Installment acceleration amounts
C - Excess compensation amounts
S - Excess shareholder payment amounts
M - Mergers and acquisitions
E - Elections to use an alternative amortization schedule
N - Notification to participants, beneficiaries, and the PBGC
CP - Eligible charity plans
R - Reporting requirements
T - Transition rules
G. GENERAL RULES
Q G-1: For which plan years can the sponsor of a single-employer defined
benefit pension plan elect to use an alternative amortization schedule under
§ 430(c)(2)(D), as added by section 201 of PRA 2010?
A G-1: (a) In general, an alternative amortization schedule under § 430(c)(2)(D)
may be elected for one or two of the plan years beginning in 2008, 2009, 2010,
or 2011, as long as the deadline for the minimum required contribution for the
plan year occurs on or after June 25, 2010. Pursuant to § 430(j)(1), this deadline
is 8½ months after the end of the plan year. Therefore, in general, plan years

3

ending on or after October 10, 2009, and beginning before January 1, 2012
(eligible plan years) are eligible for this relief.
(b) However, for plans described in section 106 of PPA ’06, the election to use an
alternative amortization schedule under § 430(c)(2)(D) can only be made for a
plan year beginning in 2011, which is the first year for which these plans are
covered by § 430. See section 202 of PRA 2010 for similar rules pertaining to
funding relief for plans described in sections 104 through 106 of PPA ’06 with
respect to plan years to which § 430 does not apply.
Q G-2: What rules generally apply to the alternative amortization schedules?
A G-2: (a) In lieu of the otherwise applicable amortization schedule for a shortfall
amortization base established for a plan year that is an eligible plan year as
defined in Q&A G-1 of this notice, a plan sponsor may elect to apply either of two
alternative amortization schedules to the shortfall amortization base: the
2 plus 7-year amortization schedule described in Q&A G-3 of this notice, or the
15-year amortization schedule described in Q&A G-4 of this notice. A plan year
for which such an election is made is known as an election year under
§ 430(c)(2)(D)(i). If an election is made to use an alternative amortization
schedule for two eligible plan years, the same schedule must be elected for both
years.
(b) In certain circumstances, the amortization installments under an alternative
amortization schedule must be increased, as described in section III.I of this
notice relating to installment acceleration amounts.
(c) See Q&A R-5 of this notice for a description of how to report the shortfall
amortization installments in years affected by an election to use an alternative
amortization schedule.
Q G-3: How are the installment amounts for a shortfall amortization base
calculated under the 2 plus 7-year amortization schedule?
A G-3: (a) If an election is made to apply the 2 plus 7-year amortization schedule
to a shortfall amortization base, the installment for each of the first two years is
determined by multiplying the amount of the shortfall amortization base
established for the election year by the effective interest rate for the plan for the
election year. The installment for each of the remaining 7 years is the level
amount calculated so that the present value of the 9 installments as of the
valuation date for the election year equals the amount of the shortfall
amortization base established for the election year.
(b) The present value of the 9 installments is determined using the segment rates
or rates from the full yield curve used to determine the target normal cost (or the
funding target, if the target normal cost is zero) for the election year. See

4

§ 1.430(h)(2)-1(e) of the Income Tax Regulations for a description of the interest
rates that may be used for this purpose, and § 1.430(h)(2)-1(f)(2) for rules
regarding the use of segment rates to determine the amount of shortfall
amortization installments.
Example G-1: (a) Assume that the sponsor of a plan with a calendar year plan
year and a January 1 valuation date elects to amortize the shortfall amortization
base of $1,000,000 established for the 2010 plan year using the 2 plus 7-year
amortization schedule. The first and second segment rates used to determine
the target normal cost for the 2010 plan year are 4.81% and 6.69%, respectively,
and the effective interest rate for the plan for the 2010 plan year is 6.00%.
(b) Each of the shortfall amortization installments for the 2010 and 2011 plan
years is $60,000, determined by multiplying the amount of the shortfall
amortization base by the effective interest rate for the plan for the 2010 plan year
($1,000,000 x 6% = $60,000). After taking into account these installments, the
remaining shortfall amortization base is equal to the amount of the shortfall
amortization base, minus the first two installments adjusted to the
January 1, 2010 valuation date using the first segment rate of 4.81%, or
$882,754 ($1,000,000 - $60,000 - ($60,000 ÷ 1.0481)). The shortfall
amortization installment for each of the next 7 plan years (2012 through 2018) is
$168,458, determined as the level amount necessary to amortize the remaining
balance of $882,754 using the first segment rate of 4.81% for the shortfall
amortization installments for 2012 through 2014, and the second segment rate of
6.69% for the shortfall amortization installments for 2015 through 2018. The total
present value of all 9 payments is $1,000,000, calculated using the first segment
rate of 4.81% for installments due for plan years 2010 through 2014 and the
second segment rate of 6.69% for plan years 2015 through 2018.
Q G-4: How are the installment amounts for a shortfall amortization base
calculated under the 15-year amortization schedule?
A G-4: If an election is made to apply the 15-year amortization schedule to a
shortfall amortization base, the installment amount for each plan year (i.e., the
election year and the subsequent 14 plan years) is the level amount needed to
amortize the shortfall amortization base established for the election year over a
period of 15 years. A shortfall amortization base for which relief is elected is
amortized using the segment rates or rates from the full yield curve used to
determine the target normal cost (or the funding target, if the target normal cost is
zero) for the election year. See § 1.430(h)(2)-1(e) for a description of the interest
rates that may be used for this purpose, and § 1.430(h)(2)-1(f)(2) for rules
regarding the use of segment rates to determine the amount of shortfall
amortization installments.
Example G-2: The facts are the same as in Example G-1, except that the plan
sponsor elects to use the 15-year amortization schedule. The shortfall

5

amortization installment due for each of the 15 plan years from 2010 through
2024 is $99,394. The shortfall amortization installment is determined using the
first segment rate of 4.81% for the installments due for plan years 2010 through
2014 and the second segment rate of 6.69% for the installments due for plan
years 2015 through 2024.
Q G-5: How does the calculation of shortfall amortization installments change if
the valuation date is not the first day of the plan year?
A G-5: (a) The shortfall amortization installments are calculated using the same
principles as the installments under the corresponding amortization schedule for
plans with valuation dates on the first day of the plan year. For example, the
amortization installments for the first two years of the 2 plus 7-year amortization
schedule are equal to the product of the shortfall amortization base and the
plan’s effective interest rate for the election year, without further adjustment for
interest.
(b) Each installment, regardless of whether the 2 plus 7-year or the 15-year
amortization schedule is used, is assumed to be paid on the valuation date when
determining the amount of the shortfall amortization installments.
Q G-6: How is an election to use an alternative amortization schedule applied to
a multiple employer plan?
A G-6: (a) In the case of a multiple employer plan to which § 413(c)(4)(A)
applies, the rules of § 430 and this notice are applied separately for each
employer under the plan, as if each employer maintained a separate plan.
Accordingly, the rules of this section apply to each plan sponsor separately, and
a plan sponsor may independently elect to use an alternative amortization
schedule for up to two eligible years with respect to the portion of the plan
attributable to that sponsor. Other sponsors of the multiple employer plan may
elect to use a different alternative amortization schedule for different eligible plan
years, or may decide not to use an alternative amortization schedule.
(b) In the case of a multiple employer plan to which § 413(c)(4)(A) does not
apply, the rules of § 430 are applied as if all participants in the plan were
employed by a single employer. Therefore, if an election is made to use an
alternative amortization schedule, such election applies to the entire plan and the
rules of this notice apply to the entire plan.
Q G-7: Will an election to use an alternative amortization schedule for a plan
affect the plan sponsor's ability to obtain a funding waiver for that plan?
A G-7: Each request for a funding waiver is reviewed based on the facts and
circumstances applying to that individual plan. See section 2.03 of Rev. Proc.
2004-15 (2004-1 C.B. 490). One relevant factor is whether the combination of a

6

funding waiver and an election to use an alternative amortization schedule would
reduce the minimum required contributions to a point where the granting of the
waiver would be adverse to the interest of plan participants in the aggregate. To
ensure that the granting of the waiver is not adverse to the interest of participants
in the aggregate, the Service may impose additional requirements relating to any
election of an alternative amortization schedule as a condition for granting a
funding waiver.
I. INSTALLMENT ACCELERATION AMOUNTS
Section 430(c)(7)(A) provides that if there is an installment acceleration amount
with respect to a plan for any plan year in the restriction period with respect to a
plan year for which an alternative amortization schedule is elected, then the
shortfall amortization installment otherwise determined under § 430(c)(2)(D) is
increased by the installment acceleration amount, subject to the limitation under
§ 430(c)(7)(B). Section § 430(c)(7)(F)(ii) defines the restriction period as the
3-plan-year period (in the case of a plan using the 2 plus 7-year amortization
schedule) or the 5-plan-year period (in the case of a plan using the 15-year
amortization schedule) beginning with the later of the election year or the first
plan year beginning after December 31, 2009. Installment acceleration amounts
for a plan year which exceed the limitation are carried over to the following plan
year if that year is within the period described in § 430(c)(7)(C)(iii)(III). See Q&A
I-6 of this notice.
Section 430(c)(7)(C) defines an installment acceleration amount with respect to a
plan year within the restriction period. In general, the installment acceleration
amount is equal to the sum of the aggregate amount of excess employee
compensation determined under § 430(c)(7)(D) (referred to in this notice as the
excess compensation amount) and the aggregate amount of dividends and
redemptions determined under § 430(c)(7)(E) (referred to in this notice as the
excess shareholder payment amount).
Under § 430(c)(7)(C)(ii), the installment acceleration amount applied to a shortfall
amortization installment for any plan year (referred to in this notice as the
acceleration adjustment) is limited to the excess (if any) of the sum of the
shortfall amortization installments for the plan year and all preceding plan years
in the amortization period, determined without regard to the election of an
alternative amortization schedule, over the sum of the shortfall amortization
installments for such plan year and all such preceding plan years, determined
after application of the alternative amortization schedule. Section 430(c)(7)(B)(ii)
further provides that if an acceleration adjustment applies, subsequent shortfall
amortization installments are reduced (beginning with the last payment due) so
that the present value of the adjusted amortization schedule is equal to the
present value of the remaining unamortized shortfall amortization base. Thus,
the effect of an installment acceleration adjustment is to accelerate, rather than
increase, the installments otherwise required.

7

Q I-1: What are the general rules for installment acceleration amounts?
A I-1: (a) For any plan year that is within the restriction period described in Q&A
I-2, an installment acceleration amount is calculated for a shortfall amortization
base for which an alternative amortization schedule has been elected. The
installment acceleration amount for a plan year is determined as the sum of the
excess compensation amount as described in section III.C of this notice and the
excess shareholder payment amount as described in section III.S of this notice.
(b) The amount added to the shortfall amortization installment for a plan year is
based on the installment acceleration amount but is limited as described in Q&A
I-4 of this notice. If the limitation described in Q&A I-4 of this notice is exceeded,
the excess amounts are carried over to subsequent plan years in accordance
with Q&A I-5 and Q&A I-6 of this notice. No amounts are added to the shortfall
amortization installments on account of installment acceleration amounts after
the period described in Q&A I-6.
(c) For any year in which an amount is added under paragraph (b) of this Q&A
I-1, the remaining shortfall amortization installments for the affected shortfall
amortization base are reduced as described in Q&A I-3, so that the present value
of the future shortfall amortization installments is the same after reflecting the
acceleration adjustment as determined disregarding the increase for the
installment acceleration amount.
(d) If an alternative amortization schedule has been elected for more than one
plan in the plan sponsor’s controlled group (within the meaning of § 412(d)(3)),
the installment acceleration amount is allocated among all plans (i) for which the
plan sponsor has elected to use an alternative amortization schedule and (ii) for
which the current plan year falls within the restriction period with respect to the
election year. The rules related to the allocation of the installment acceleration
amount are described in Q&A I-7 and Q&A I-8 of this notice. See Q&A I-9 and
Q&A I-10 of this notice for additional considerations if the plan sponsor has
elected to use an alternative amortization schedule for more than one election
year for one or more plans in the controlled group.
(e) Quarterly installments required under § 430(j)(3) are determined without
regard to any increase due to an installment acceleration amount, whether the
required annual payment for a plan year is based on 90% of the minimum
required contribution for that plan year under § 430(j)(3)(D)(ii)(I) or 100% of the
minimum required contribution for the prior plan year under § 430(j)(3)(D)(ii)(II).
Q I-2: What is the restriction period with respect to an election year?
A I-2: (a) If the plan sponsor elects to use the 2 plus 7-year amortization
schedule for a plan year, the restriction period with respect to that election year is

8

the 3-year period beginning with the later of the election year or the first plan year
beginning after December 31, 2009. If the plan sponsor elects to use the 15-year
amortization schedule for a plan year, the restriction period with respect to that
election year is the 5-year period beginning with the later of the election year or
the first plan year beginning after December 31, 2009.
(b) For example, if the sponsor of a calendar-year plan elects to use the 2 plus
7-year amortization schedule for the shortfall amortization base established for
the plan year beginning in 2009 (the 2009 election year), the restriction period
would be the 3-year period beginning January 1, 2010, and ending
December 31, 2012. If instead, the plan sponsor elects to use the 15-year
amortization schedule with a 2010 election year, the restriction period would be
the 5-year period beginning January 1, 2010, and ending December 31, 2014.
Q I-3: How does the acceleration adjustment affect shortfall amortization
installments for future years?
A I-3: Installment acceleration amounts are intended to accelerate the timing of
shortfall amortization installments, not increase the total amount of installments
associated with a shortfall amortization base. When an acceleration adjustment
is added to the amortization installment for a shortfall amortization base for a
plan year, the subsequent shortfall amortization installments are reduced to the
extent necessary so that the present value of the remaining shortfall amortization
installments for that base (including the installment for the current year) is the
same as the present value of the remaining shortfall amortization installments for
that base before the increase for the acceleration adjustment. Under
§ 430(c)(7)(B)(ii), these reductions are applied in reverse order, beginning with
the last shortfall amortization installment due for the shortfall amortization base.
For this purpose, the present value of the remaining payments is determined
using the segment rates or the full yield curve used to determine the target
normal cost (or the funding target, if the target normal cost is zero) for the year
for which the acceleration adjustment is added to the shortfall amortization
installment.
Example I-1. (a) The facts are the same as in Example G-1, except that an
acceleration adjustment of $214,000 is added to the shortfall amortization
installment for the 2012 plan year for the shortfall amortization base for which the
alternative amortization schedule was elected. Assume for purposes of this
example that the first and second segment rates used to calculate the target
normal cost for the 2012 plan year are 5.50% and 6.25%, respectively, and that
there were no installment acceleration amounts for any previous plan years.
(b) The shortfall amortization installment for the 2012 plan year, increased to
reflect the acceleration adjustment, is $382,458. This is equal to the
otherwise-applicable shortfall amortization installment of $168,458 under the

9

2 plus 7-year amortization schedule, plus the acceleration adjustment of
$214,000.
(c) Prior to applying the acceleration adjustment, seven shortfall amortization
installments of $168,458 each remain as of January 1, 2012. Using the segment
rates for the 2012 plan year, the present value of these remaining installments is
$1,000,426.
(d) The subsequent amortization installments are reduced or eliminated as
required under § 430(c)(7)(B)(ii). Accordingly, the shortfall amortization
installment for the 2018 plan year is eliminated, and the installment for the 2017
plan year is reduced to $37,233. This results in the following shortfall
amortization installments, which have the same present value ($1,000,426) as
the remaining installments prior to applying the acceleration adjustment:
Year
2012
2013
2014
2015
2016
2017
2018
Present value as of
January 1, 2012

Shortfall amortization installments
Before adjustment After adjustment
$168,458
$382,458
168,458
168,458
168,458
168,458
168,458
168,458
168,458
168,458
168,458
37,233
168,458
0
$1,000,426

$1,000,426

Q I-4: How is the § 430(c)(7)(C)(ii) annual limitation on the increase in the
shortfall amortization installment applied?
A I-4: (a) The annual limitation on the increase in the shortfall amortization
installment is determined for a shortfall amortization base as the excess of (i) the
sum (without interest) of the shortfall amortization installments for the plan year
and all preceding plan years, determined as if the sponsor had not elected the
alternative amortization schedule, over (ii) the sum (without interest) of the actual
shortfall amortization installments for the plan year and all preceding plan years,
reflecting the alternative amortization schedule elected by the plan sponsor. The
shortfall amortization installments in clause (ii) of the preceding sentence reflect
any acceleration adjustments for all preceding plan years, but not for the current
plan year. Thus, as of the end of the plan year for which a shortfall amortization
installment for a shortfall amortization base is increased, the cumulative amount
of the shortfall amortization installments for that base, including any increase on
account of an installment acceleration amount, will not be greater than the
cumulative amount of the shortfall amortization installments for that base
determined as if the alternative amortization schedule had not been elected.

10

In addition, the increase is limited so that it does not cause the increased shortfall
amortization installment for that plan year to exceed the present value of the
remaining shortfall amortization installments for that base, determined as of the
valuation date (without regard to the increase attributable to the installment
acceleration amount for the plan year).
(b) The annual limitation is applied separately to the increase in the shortfall
amortization installment for each affected shortfall amortization base. The
resulting increase in the shortfall amortization installment for a base is not
affected by whether or not the increases in the shortfall amortization installments
for any other affected base exceed the annual limitation for that base.
Example I-2: (a) The facts are the same as in Example I-1, except that the
installment acceleration amount determined for the 2012 plan year is $250,000.
(b) Based on the first and second segment rates for the 2010 election year of
4.81% and 6.69% respectively, each shortfall amortization installment would
have been $167,698 under the 7-year amortization schedule that would have
applied absent an election to use an alternative amortization schedule. The
annual limitation on the installment acceleration amount is the excess of (1) the
sum of shortfall amortization installments under the 7-year amortization schedule
that would have applied absent the election to use an alternative amortization
schedule, over (2) the sum of the shortfall amortization installments under the
2 plus 7-year amortization schedule elected by the plan sponsor, or $214,636, as
shown in the table below:
Shortfall amortization installments
Without alternative
Reflecting alternative
Year
amortization schedule
amortization schedule
(a)
(b)
2010
$167,698
$60,000
2011
167,698
60,000
2012
167,698
168,458
Total
$503,094
$288,458
Excess of (a) over (b)
$214,636
(c) Using the first and second segment rates for the 2012 plan year of 5.50% and
6.25%, respectively, the present value of the remaining shortfall amortization
installments prior to application of the installment acceleration amount for 2012 is
$1,000,426. The increase in the shortfall amortization installment for the 2012
plan year is limited to the smaller of this amount or the annual limitation in
§ 430(c)(7)(C)(ii), or $214,636. The increased shortfall amortization installment
for 2012 for the affected shortfall amortization base is therefore $168,458 +
$214,636, or $383,094.

11

Q I-5: What carryover rules apply if the otherwise-applicable increase in the
shortfall amortization installment for an amortization base exceeds the annual
limitation described in Q&A I-4 of this notice?
A I-5: (a) If the otherwise-applicable increase in the shortfall amortization
installment for a shortfall amortization base exceeds the annual limitation
described in Q&A I-4 of this notice for that base for a plan year, the excess
(referred to in this notice as an excess installment acceleration amount) is carried
over and added to the increase in the shortfall amortization installment for that
base for the following plan year if that plan year is within the carryover period
described in Q&A I-6 of this notice. Any carryover of an excess installment
amortization amount is added only to the installments for the shortfall
amortization base to which it was originally attributed.
(b) The acceleration adjustment added to the shortfall amortization installment for
any plan year within the carryover period described in Q&A I-6 of this notice is
equal to (i) the installment acceleration amount for that shortfall amortization
base, if that base is within the restriction period, plus (ii) any installment
acceleration amount carried over from previous years in accordance with this
Q&A I-5, with that sum limited by the annual limitation in Q&A I-4 of this notice.
Example I-3. (a) The facts are the same as in Example I-2. The amount carried
over to the 2013 plan year is equal to $35,364, which is the excess of the
installment acceleration amount determined for the 2012 plan year, or $250,000,
over the amount that was added to the shortfall amortization installment for the
2012 plan year after application of the annual limitation, or $214,636. No
installment acceleration amount is applicable for the 2013 plan year, because the
2013 plan year is not within the restriction period for the shortfall amortization
base established for the 2010 plan year.
(b) Before application of the annual limit under § 430(c)(7)(C)(ii) and Q&A I-4 of
this notice, the shortfall amortization installment for 2013 would be increased to
$168,458 + $35,364, or $203,822. The annual limitation for the 2013 plan year is
the excess of (i) the sum of the shortfall amortization installments for the 2010
through 2013 plan years, determined as if the sponsor had not elected the
alternative amortization schedule, over (ii) the sum of the actual shortfall
amortization installments for the 2010 through 2013 plan years reflecting the
alternative amortization schedule elected by the plan sponsor, including
acceleration adjustments added to the shortfall amortization installments through
2012. Accordingly, the annual limitation for the 2013 plan year is $0, as shown in
the table below:

12

Shortfall amortization installments
Without alternative
Reflecting alternative
Year
amortization schedule
amortization schedule
(a)
(b)
2010
$167,698
$60,000
2011
167,698
60,000
2012
167,698
383,094
2013
167,698
168,458
Total
$670,792
$671,552
Excess of (a) over (b)
$0
Q I-6: For how long will excess installment acceleration amounts be carried
over?
A I-6: (a) In general, any excess installment acceleration amount for a shortfall
amortization base is carried over each plan year until there is no longer an
excess installment acceleration amount for the associated shortfall amortization
base or until the associated shortfall amortization base is completely amortized.
However, in no event is an excess installment acceleration amount carried over
to a plan year which begins after the first plan year following the last plan year in
the restriction period for a base being amortized using the 2 plus 7-year
amortization schedule. For a base being amortized using the 15-year
amortization schedule, no excess installment acceleration amount is carried over
to a plan year which begins after the second plan year following the last plan
year in the restriction period for that base.
(b) For example, if the sponsor of a plan with a calendar-year plan year elected
to amortize the shortfall amortization base established for the 2010 plan year
using the 2 plus 7-year amortization schedule, the restriction period would end in
2012 (the end of the 3-year period beginning with the election year of 2010) and
the last year for which any installment acceleration amounts would be carried
over from previous years is the following year, 2013. Any excess installment
acceleration amounts remaining at the end of 2013 would not be added to the
remaining shortfall amortization installments for that base (or any other base) for
any future years.
(c) If, instead, the plan sponsor elected to amortize the shortfall amortization
base established for 2010 using the 15-year amortization schedule, the
restriction period would end in 2014 (the end of the 5-year period beginning with
the election year of 2010) and the last year for any installment acceleration
amounts to be carried over from previous years is the second year following the
end of the restriction period, or 2016. Any excess installment acceleration
amounts remaining at the end of 2016 would not be added to the remaining

13

shortfall amortization installments for that base (or any other base) for any future
years.
Q I-7: How is an installment acceleration amount for a plan year allocated
among plans for which the sponsor has elected to use an alternative amortization
schedule?
A I-7: (a) Under § 430(c)(7)(F)(iii), if a plan sponsor elects to use alternative
amortization schedules for two or more plans, the installment acceleration
amount for a plan year is allocated among those plans. If the plan sponsor has
elected to use an alternative amortization schedule for only one election year for
each plan, the installment acceleration amount for a plan year is allocated among
all plans of the plan sponsor’s controlled group for which a shortfall amortization
base is being amortized using an alternative amortization schedule, but only if
that shortfall amortization base is still in its restriction period for the plan year for
which the installment acceleration amount is being allocated. These plans are
referred to in this notice as affected plans. If the plan year for which the
installment acceleration amount is determined is later than the last year of a
restriction period for all shortfall amortization bases for a plan, that plan is not an
affected plan, and is disregarded in allocating the installment acceleration
amount for that plan year. See Q&A I-9 and Q&A I-10 of this notice for rules
regarding the allocation of installment acceleration amounts if an election to use
an alternative amortization schedule is made for more than one election year for
one or more plans in the controlled group.
(b) The first step in allocating the installment acceleration amount among the
affected plans is to identify the shortfall amortization bases for which the shortfall
amortization installments are determined using an alternative amortization
schedule, and which are still in their restriction period as described in Q&A I-2 of
this notice (affected shortfall amortization bases). The second step is to
determine, for each affected shortfall amortization base, the difference between
(i) the shortfall amortization installment without reflecting the alternative
amortization schedule, and (ii) the shortfall amortization installment for the first
year of the alternative amortization schedule, determined without regard to any
installment acceleration amount that may have applied for that first year. This
difference is referred to in this notice as the first-year reduction with respect to
the affected shortfall amortization base.
(c) The portion of the installment acceleration amount allocated to an affected
plan for a plan year (the allocated portion) is determined by multiplying the
installment acceleration amount for that plan year by a fraction, the numerator of
which is the first-year reduction for that plan, and the denominator of which is the
sum of the first-year reductions for all affected plans. Each amount so allocated
is limited as described in Q&A I-4 and Q&A I-5 of this notice, and the resulting
acceleration adjustment is added to the corresponding affected shortfall
amortization base.

14

Example I-4. (a) Assume that a plan sponsor has two plans (with calendar year
plan years) for which an alternative amortization schedule has been elected:
Plan
Election year
Alternative amortization schedule elected
Shortfall amortization installments
for first year of amortization schedule:
Without alternative schedule
Reflecting alternative schedule
First-year reduction

Plan A
2009
2 plus 7-year

$167,698
60,000
$107,698

Plan B
2011
15-year

$252,579
146,878
$105,701

Installment acceleration amounts of $100,000 and $300,000 are calculated for
the plan years beginning January 1, 2010, and January 1, 2011, respectively.
(b) The installment acceleration amount of $100,000 determined for the plan year
beginning January 1, 2010, is applied only to the shortfall amortization
installment for Plan A, and the carryover of any amount over the annual limitation
for 2010 and future years is applied only to Plan A. No portion of the installment
acceleration amount for 2010 is allocated to Plan B because it is not an affected
plan in 2010.
(c) The installment acceleration amount of $300,000 determined for the plan year
beginning January 1, 2011, is allocated ratably between Plans A and B based on
the first-year reduction for each plan. Accordingly, the allocated portion of the
installment acceleration amount for Plan A for the 2011 plan year is $151,404,
determined by multiplying the installment acceleration amount of $300,000 by a
fraction, the numerator of which is the first-year reduction for Plan A ($107,698)
and the denominator of which is the total of such first-year reductions for Plans A
and B ($107,698 + $105,701, or $213,399). Similarly, the portion of the
$300,000 installment acceleration amount for the 2011 plan year that is allocated
to Plan B is $148,596.
Q I-8: How is the installation acceleration amount determined if the amount is
required to be allocated among plans that have differing plan years?
A I-8: If an alternative amortization schedule has been elected for more than one
plan and the plans do not all have the same plan year, then installment
acceleration amounts are determined on a calendar year basis and the
installment acceleration amount so determined for a calendar year is allocated
among the plans based on the plan years that begin within that calendar year.
See Q&A S-4 of this notice for related rules.
Q I-9: How do the installment acceleration rules apply if the sponsor elected to
use an alternative amortization schedule for two election years?

15

A I-9: (a) Section 430(c)(7)(A) provides that the shortfall amortization installment
otherwise determined with respect to an election year is increased on account of
the installment acceleration amount. Section 430(c)(7) provides for an allocation
of the installment acceleration amount among plans for which the sponsor
elected to use an alternative amortization schedule (see § 430(c)(7)(F)(iii) and
Q&A I-7 and Q&A I-8 of this notice), but does not provide for the allocation of the
installment acceleration amount among individual shortfall amortization bases
within a plan. Accordingly, subject to the adjustments in Q&A I-4 through Q&A
I-6 of this notice, the increase on account of the excess compensation amount
and excess shareholder payment amount is added separately to the shortfall
amortization installment for each base for which an alternative amortization
schedule was elected.
(b) For example, if the plan sponsor elected an alternative amortization schedule
for the plan year beginning in 2009 and elected the same alternative amortization
schedule for the plan year beginning in 2010, and if the installment acceleration
amount for the 2010 plan year is $500,000, then, subject to the annual limitation
in § 430(c)(7)(C)(ii), the shortfall amortization charge for the 2010 plan year
reflects an increase of $500,000 with respect to the shortfall amortization base
established for the 2009 plan year and an additional increase of $500,000 with
respect to the shortfall amortization base established for the 2010 plan year, for a
total increase in the minimum required contribution for the 2010 plan year of
$1,000,000. Thus, the minimum required contribution can be increased by $2 for
every $1 of the installment acceleration amount if an election has been made for
two plan years and the installment acceleration amount arises in the restriction
period for both election years (but can only be increased by $1 for every $1 of the
installment acceleration amount if an election has been made for only one plan
year).
Q I-10: How is an installment acceleration amount allocated among plans for
which the sponsor has elected to use an alternative amortization schedule for
shortfall amortization bases established for more than one election year?
A I-10: (a) As discussed in Q&A I-9 of this notice, a separate installment
acceleration amount determined for a plan year is added to the shortfall
amortization installment for that plan year for each affected shortfall amortization
base. Accordingly, if a plan sponsor elects to use an alternative amortization
schedule for two shortfall amortization bases for any plan in the controlled group,
and if both of those bases are still within the restriction period (as described in
Q&A I-2 of this notice), the installment acceleration amount is added twice.
However, because no plan can have more than two election years, the
installment acceleration amount cannot be added more than twice, even if the
election years for various plans within the controlled group fall in more than two
separate plan years. Each installment acceleration amount is then allocated

16

among the plans based on the rules described in Q&A I-7 and Q&A I-8 of this
notice, as described in paragraphs (b) through (e) of this Q&A I-10.
(b) First, all affected plans in the controlled group and the affected shortfall
amortization bases are identified, as outlined in Q&A I-7 of this notice,
disregarding those which are no longer in the restriction period for the year for
which the installment acceleration amount is being allocated. The affected
shortfall amortization bases are then divided into two groups, the earlier-year
shortfall amortization bases and the later-year shortfall amortization bases. The
group of earlier-year shortfall amortization bases consists of the affected shortfall
amortization bases for each plan for which the election year is not later than the
election year for any other affected shortfall amortization base for that plan. The
group of later-year shortfall amortization bases consists of the affected shortfall
amortization bases in the group of affected shortfall amortization bases that are
not in the group of earlier-year shortfall amortization bases. Note that a
later-year shortfall amortization base for a plan will move to the group of
earlier-year shortfall amortization bases when the original earlier-year shortfall
amortization base for that plan is no longer in its restriction period.
(c) The allocated portion of the installment acceleration amount for each plan
with an earlier-year shortfall amortization base is determined by multiplying the
installment acceleration amount calculated for the current plan year by a fraction,
the numerator of which is the first-year reduction (as described in paragraph (b)
of Q&A I-7 of this notice) for the earlier-year shortfall amortization base for that
plan and the denominator of which is the sum of such first-year reductions for the
earlier-year shortfall amortization bases for all plans in the controlled group.
(d) Similarly, the allocated portion of the installment acceleration amount for each
plan with a later-year shortfall amortization base is determined by multiplying the
installment acceleration amount calculated for the current plan year by a fraction,
the numerator of which is the first-year reduction for the later-year shortfall
amortization base for that plan and the denominator of which is the sum of such
first-year reductions for all plans in the controlled group with later-year shortfall
amortization bases.
(e) The increase in the shortfall amortization installment for each affected
shortfall amortization base is subject to the adjustments described in Q&A I-4
and Q&A I-5 of this notice.
Example I-5: (a) The facts are the same as for Example I-4, except that the plan
sponsor also elects to use the 2 plus 7-year amortization schedule for Plan A for
the shortfall amortization base established for 2011:

17

Plan
Election year
Alternative amortization schedule
elected
End of restriction period
Shortfall amortization installments
for first year of amortization
schedule:
Without alternative schedule
Reflecting alternative schedule
First-year reduction

Plan A
2009
2 plus
7-year
2012

Plan A
2011
2 plus
7-year
2013

$167,698
60,000
$107,698

503,094
172,500
$330,594

Plan B
2011
15-year
2015

$252,579
146,878
$105,701

An installment acceleration amount of $100,000 is determined for the plan year
beginning January 1, 2011.
(b) Because an alternative amortization schedule has been elected for two
shortfall amortization bases for Plan A, and because the 2011 plan year is in the
restriction period for each base, an installment acceleration amount is added to
each shortfall amortization installment for 2011. The installment acceleration
amounts are allocated between Plan A and Plan B based on the rules in Q&A I-7
and Q&A I-8 of this notice. The earlier-year shortfall amortization base for Plan A
is the first shortfall amortization base for which an alternative amortization
schedule was elected for that plan (the base established for 2009). The
earlier-year shortfall amortization base for Plan B is the only base for which an
alternative amortization schedule was elected for that plan, established for the
2011 plan year.
(c) The installment acceleration amount of $100,000 is allocated between the
plans in proportion to the first-year reduction for each earlier-year shortfall
amortization base for the first year of the amortization schedule. The allocated
portion of the installment acceleration amount for Plan A is $50,468. This
amount is calculated by multiplying the installment acceleration amount of
$100,000 by a ratio, the numerator of which is the first-year reduction for the
earlier-year shortfall amortization base for Plan A and the denominator of which
is the sum of the first-year reductions for the earlier-year shortfall amortization
bases for Plans A and B ($100,000 x $107,698 ÷ ($107,698 + $105,701) =
$50,468). Similarly, the allocated portion of the installment acceleration amount
for Plan B is $49,532.
(d) In addition, a second installment acceleration amount of $100,000 is added to
the shortfall amortization installments for the later-year shortfall amortization
bases. The only later-year shortfall amortization base for which the sponsor has
elected an alternative amortization schedule is the base established for Plan A
for 2011. Therefore, the full $100,000 installment acceleration amount is
allocated to Plan A, and the total installment acceleration amount allocated to

18

both affected shortfall amortization bases for Plan A amounts to $150,468 for the
2011 plan year. As developed earlier in this example, an additional installment
acceleration amount of $49,532 is allocated to Plan B, for a total installment
acceleration amount of $200,000 for the 2011 plan year.
Example I-6: (a) The facts are the same as in Example I-5. An installment
acceleration amount of $150,000 is determined for the 2013 plan year.
(b) Because the shortfall amortization base established for Plan A for the 2009
plan year is no longer in the restriction period, that base is ignored when
determining the installment acceleration amounts that are allocated to that plan
for that plan year. Accordingly, the shortfall amortization base established for
2011 becomes the earlier-year shortfall amortization base for Plan A, and there
are no later-year shortfall amortization bases for either plan.
(c) Because neither plan has more than one shortfall amortization base for which
an alternative amortization schedule was elected and which is still in its
restriction period, the installment acceleration amount is only applied once, and
the amount is allocated between Plans A and B based on the reduction in the
first-year installments for each base. The allocated portion of the installment
acceleration amount for Plan A for the 2013 plan year is $113,660. This amount
is calculated by multiplying the installment acceleration amount for the 2013 plan
year ($150,000) by a ratio, the numerator of which is the first-year reduction for
the shortfall amortization base established for Plan A for 2011 and the
denominator of which is the sum of the first-year reductions for the shortfall
amortization bases established for Plan A and Plan B for that same year
($150,000 x $330,594 ÷ ($330,594 + $105,701) = $113,660). Similarly, the
allocated portion of the installment acceleration amount for the shortfall
amortization base established for Plan B for 2011 is $36,340.
Q I-11: How do the rules of this section apply to multiple employer plans?
A I-11: (a) In the case of a multiple employer plan to which § 413(c)(4)(A)
applies, the rules of § 430 and this section are applied separately for each
employer under the plan as if each employer maintained a separate plan.
Accordingly, any installment acceleration amount is determined separately for
each plan sponsor who elects an alternative amortization schedule, based on the
excess compensation amount for employees in that sponsor’s controlled group in
accordance with the rules of section III.C of this notice, and on the excess
shareholder payment amounts for that sponsor’s controlled group determined in
accordance with the rules of section III.S of this notice. The resulting installment
acceleration amount for a plan sponsor is added only to shortfall amortization
bases for which that sponsor elected to use an alternative amortization schedule,
in accordance with the rules of this section.

19

(b) In the case of a multiple employer plan to which § 413(c)(4)(A) does not
apply, the rules of § 430 are applied as if all participants in the plan were
employed by a single employer. Therefore, if an election is made to use an
alternative amortization schedule, the installment acceleration amount is
determined based on the excess compensation amount for all employees of the
controlled groups for all sponsoring employers in accordance with section III.C of
this notice, and on the excess shareholder payment amounts for any member of
the controlled groups for all sponsoring employers, in accordance with section
III.S of this notice.
C. EXCESS COMPENSATION AMOUNTS
The excess employee compensation amount that is included in the installment
acceleration amount is defined at § 430(c)(7)(D). In general, the excess
employee compensation amount is, with respect to any employee for any plan
year, the excess (if any) over $1,000,000, of the aggregate amount includible in
income for remuneration during the calendar year in which the plan year begins
for services performed by the employee for the plan sponsor (whether or not
performed during the calendar year). Under § 430(c)(7)(D)(vi), the term
“employee” includes a self-employed individual who is treated as an employee
under § 401(c), and the term “compensation” includes earned income of such
individual with respect to such self-employment. Pursuant to § 430(c)(7)(D)(viii),
the $1,000,000 amount is indexed under § 1(f)(3) for calendar years after 2010,
rounded to the next lowest multiple of $1,000.
Under § 430(c)(7)(D)(ii), if during any calendar year, assets are set aside or
reserved (directly or indirectly) in a trust (or other arrangement as determined by
the Secretary), or transferred to such a trust or other arrangement, by a plan
sponsor for purposes of paying deferred compensation of an employee under a
nonqualified deferred compensation plan (as defined in § 409A) of the plan
sponsor, then, for purposes of determining the excess employee compensation
amount with respect to an employee, the amount of those assets is treated as
remuneration of the employee includible in income for the calendar year, unless
that amount is taken into account under § 430(c)(7)(D)(i) for that year. An
amount that is taken into account as remuneration includible in income under
§ 430(c)(7)(D)(ii) for a calendar year is not taken into account under
§ 430(c)(7)(D) for any subsequent calendar year. Sections 430(c)(7)(D)(iii)
through (v) provide four additional exceptions from the remuneration taken into
account for purposes of calculating the excess employee compensation amount.
Q C-1: How is the excess compensation amount that is included in the
installment acceleration amount determined?
A C-1: (a) The excess compensation amount for a plan year is the sum, for all
employees of the plan sponsor, of the portion of the PRA compensation amount
for each such employee that exceeds $1,000,000 (as indexed for changes in the

20

cost-of-living index). For this purpose, the PRA compensation amount for an
employee for a plan year is determined as the employee’s compensation
amount, as described in Q&A C-2 of this notice, adjusted by adding any set-aside
amount as described in Q&A C-3 of this notice, but disregarding any amounts
excluded as described in Q&A C-4 through Q&A C-7 of this notice.
(b) For purposes of determining the excess compensation amount, (1) the term
“employee” includes former employees and self-employed individuals who are
treated as employees under § 401(c), and (2) the term “plan sponsor” includes all
members of the plan sponsor’s controlled group (as defined in § 412(d)(3)).
However, in the case of a plan established or maintained jointly by an employer
and an employee organization, the term “plan sponsor” includes all members of
the employer’s controlled group (as defined in § 412(d)(3)).
Q C-2: What is the compensation amount for an employee for a plan year?
A C-2: The compensation amount for an employee for a plan year is equal to the
amount that is includible in the employee’s income for the calendar year in which
the plan year begins and that constitutes remuneration for services performed by
the employee for the plan sponsor (including remuneration for services
performed by the employee for the plan sponsor in earlier years that is includible
in the employee’s income for the calendar year in which the plan year begins,
subject to the special rules described in Q&A C-3 through Q&A C-7 of this
notice). With respect to a self-employed individual who is treated as an
employee under § 401(c), compensation for this purpose includes the earned
income of that individual with respect to such self-employment for the taxable
year ending during the calendar year in which the plan year begins.
Q C-3: How is the set-aside amount determined for an employee for a plan
year?
A C-3: (a) The set-aside amount for an employee for a plan year that is added to
the compensation amount under Q&A C-2 of this notice for purposes of
determining the PRA compensation amount is equal to the fair market value of
assets that are set aside or reserved (directly or indirectly) in a trust (or other
arrangement as determined by the Secretary), or transferred to such a trust or
other arrangement, by a plan sponsor for purposes of paying deferred
compensation of the employee under a nonqualified deferred compensation plan
(as defined in § 409A) during the calendar year in which the plan year (of the
qualified plan) begins, except to the extent that amount is includible in income for
that year. For this purpose, any guidance under § 409A(b)(1) concerning assets
that are set aside or reserved (directly or indirectly) in a trust or other
arrangement (or transferred to such a trust or other arrangement) also applies for
purposes of § 430(c)(7)(D)(ii) (but without regard to the situs of the trust or other
arrangement).

21

(b) An amount that is taken into account as part of the set-aside amount for an
employee for a plan year is not also taken into account in determining the
employee’s compensation amount under Q&A C-2 of this notice for any
subsequent period.
(c) The fair market value of assets set aside, reserved, or transferred is
determined at the time they are set aside, reserved, or transferred. If such
assets are permitted under the trust or arrangement to be applied to the deferred
compensation of more than one employee, those assets are included in the
set-aside amounts of each employee proportionately to the aggregate present
value of each employee’s benefits for which assets are permitted to be applied
under the trust or arrangement.
(d) Payments made by a foreign corporation to a foreign trust (or other foreign
arrangement as determined by the Secretary) on behalf of a nonresident alien
are not considered to be set-aside amounts for the purpose of determining
excess compensation under § 430(c)(7)(D)(i) to the extent that such payments
would not have been subject to U.S. income tax as income effectively connected
with the conduct of a trade or business within the United States if they had been
paid to the nonresident alien in cash.
(e) The exceptions set forth in §§ 430(c)(7)(iii) through (v), as described in Q&A
C-4 through Q&A C-7 of this notice, do not apply for purposes of determining the
set-aside amount under § 430(c)(7)(ii), as described in this Q&A C-3.
Q C-4: Section 430(c)(7)(iii) provides that remuneration taken into account under
§ 430(c)(7)(D)(i) only includes remuneration to the extent attributable to services
performed by the employee for the plan sponsor after February 28, 2010. How is
this rule applied?
A C-4: Remuneration included in an employee’s compensation amount under
Q&A C-2 of this notice for a calendar year that is attributable to services
performed before March 1, 2010, is subtracted from the amount otherwise
included in an employee’s remuneration in determining the compensation
amount for that calendar year. The rules of Q&A-23 of Notice 2009-8 (2009-4
I.R.B. 347) apply for purposes of determining whether a compensation amount
under Q&A C-2 of this notice for a calendar year is attributable to services
performed before March 1, 2010 (applied by substituting periods before March 1,
2010, for periods before January 1, 2009). However, for remuneration which is
not directly attributable to services performed during specific months within the
2010 calendar year, the remuneration is treated as attributable pro rata to
months during 2010. Thus, for a $12,000 bonus payment made to an employee
for services rendered by the employer for the plan sponsor during the entire 2010
calendar year, $2,000 would be attributable to services performed during January
and February of 2010, and only the remaining $10,000 would be included in
remuneration for purposes of § 430(c)(7)(D)(i).

22

Q C-5: Section 430(c)(7)(D)(iv) provides that remuneration taken into account
under § 430(c)(7)(D)(i) does not include any amount includible in income with
respect to the granting, after February 28, 2010, of service recipient stock (within
the meaning of § 409A) that, upon such grant, is subject to a substantial risk of
forfeiture (as defined in § 83(c)(1)) for at least 5 years from the date of that grant.
How is this rule applied?
A C-5: For purposes of determining whether a grant of service recipient stock
made after February 28, 2010, is taken into account under § 430(c)(7)(D)(i),
except as provided in the final sentence of this Q&A C-5, the determination of
whether a risk of forfeiture constitutes a substantial risk of forfeiture (as defined in
§ 83(c)(1)) for at least 5 years from the date of that grant is made as of the date
of grant. Thus, provided the risk of forfeiture with respect to such a grant is a
substantial risk of forfeiture as of the date of grant, the grant does not fail to meet
the requirement that the service recipient stock be subject to a substantial risk of
forfeiture for at least 5 years from the date of that grant merely because the risk
of forfeiture lapses prior to the end of the 5-year period pursuant to the terms of
the grant. For example, if the terms of the grant on the date of the grant impose
a substantial risk of forfeiture for at least 5 years, but the terms provide for the
risk to lapse if the grantee dies, the grant will be treated as being subject to a
substantial risk of forfeiture for at least 5 years from the date of the grant for
purposes of § 430(c)(7)(D) even if the risk of forfeiture lapses within 5 years after
the grant due to the death of the grantee. However, if any amendment is made
to the terms of the grant after February 28, 2010, that causes the grant to cease
to be subject to a substantial risk of forfeiture (as defined in § 83(c)(1)) for at
least 5 years from the date of that grant, the special rule described in this Q&A
C-5 will cease to apply to the grant.
Q C-6: Section 430(c)(7)(D)(v)(I) provides that the remuneration taken into
account under § 430(c)(7)(D)(i) does not include any remuneration payable on a
commission basis solely on account of income directly generated by the
individual performance of the individual to which that remuneration is payable.
How is this rule applied?
A C-6: For purposes of § 430(c)(7)(D)(i)(I), remuneration is payable on a
commission basis only if the facts and circumstances show that the remuneration
is paid solely on account of income generated directly by the individual
performance of the individual to whom the compensation is paid and the income
is a result of a direct sale of a product or service to an unrelated customer in the
ordinary course of the business of the employer. Thus, remuneration is not
payable on a commission basis if the remuneration is paid on account of broader
performance standards, such as on account of the income produced by a
business unit of the employer or on account of the disposition of a business unit
that is not in the ordinary course of business of the employer. However,
remuneration does not fail to be attributable directly to the individual merely

23

because support services, such as secretarial or research services, are utilized in
generating the income.
Q C-7: Section 430(c)(7)(D)(v)(II) provides that the remuneration taken into
account under § 430(c)(7)(D)(i) does not include any remuneration consisting of
nonqualified deferred compensation, restricted stock, stock options, or stock
appreciation rights payable or granted under a written binding contract that was
in effect on March 1, 2010, and that was not modified in any material respect
before that remuneration is paid. How is this rule applied?
A C-7: (a) Remuneration consisting of nonqualified deferred compensation,
restricted stock, stock options, or stock appreciation rights payable or granted
under a written binding contract that was in effect on March 1, 2010, and that
was not modified in any material respect before that remuneration is paid and
which is included in an employee’s compensation amount under Q&A C-2 of this
notice for a plan year is subtracted from the amount otherwise included in
determining the PRA compensation amount for that plan year. For this purpose,
remuneration is not payable or granted under a written binding contract that is in
effect on March 1, 2010, if the employee does not have a legally binding right to
the remuneration on March 1, 2010, under the rules set forth in § 1.409A-1(b)(1).
Whether a material modification has occurred is determined under rules similar to
the rules under § 1.409A-6(a)(4) (substituting March 1, 2010, for October 3,
2004, or January 1, 2005), as applied based on the nonqualified deferred
compensation, restricted stock, stock options, or stock appreciation rights
modified. For example, if an employee’s contract addresses both cash
compensation and stock options, and a material modification is made on or after
March 1, 2010, with respect to the employee’s cash compensation, but the
modification does not affect the employee’s stock options, then the compensation
in the form of stock options is not considered to have been materially modified.
(b) For purposes of applying § 430(c)(7)(D)(v)(II), nonqualified deferred
compensation does not include remuneration that is not deferred for more than a
brief period of time after the end of the employer's taxable year, as described in
§ 1.404(b)-1T, Q&A-2. Accordingly, compensation is not considered to consist of
nonqualified deferred compensation for purposes of the exception described in
this Q&A C-7 to the extent that such compensation is received on or before the
15th day of the 3rd calendar month after the end of the employer's taxable year
in which the related services are rendered.
S. EXCESS SHAREHOLDER PAYMENT AMOUNTS
Section 430(c)(7)(E) defines the excess shareholder payment amount that is
included in the installment acceleration amount. The excess shareholder
payment amount for a plan year is the excess (if any) of the sum of dividends
declared during the plan year by the plan sponsor plus the aggregate amount
paid for the redemption of stock of the plan sponsor redeemed during the plan

24

year, over the greater of two amounts. The first such amount is the adjusted net
income (within the meaning of section 4043 of ERISA) of the plan sponsor for the
preceding plan year, determined without regard to any reduction by reason of
interest, taxes, depreciation, or amortization. The second such amount is, in the
case of a plan sponsor that determined and declared dividends in the same
manner for at least 5 consecutive years immediately preceding such plan year,
the aggregate amount of dividends determined and declared for such plan year
using such manner. Sections 430(c)(7)(E)(ii) through (v) provide certain
exceptions from dividends and redemptions that are taken into account in
determining the excess shareholder payment amount.
Q S-1: How is the excess shareholder payment amount that is included in the
installment acceleration amount determined?
A S-1: (a) Except as described in this Q&A S-1 and Q&A S-5 through Q&A S-8
of this notice, the excess shareholder payment amount for a plan year is the
excess (if any) of (i) the sum of dividends declared by the plan sponsor during
the plan year for which the installment acceleration amount is calculated
(regardless of whether the dividends are paid after the last day of that plan year)
plus the aggregate amount paid for the redemption of stock (as described in
§ 317(b)) of the plan sponsor that occurs during such plan year (regardless of
whether the redemption was announced before the first day of that plan year),
over (ii) the adjusted net income for the prior plan year as described in Q&A S-2
of this notice. However, in the case of a plan sponsor that determined and
declared dividends in the same manner for at least 5 consecutive years, as
described in Q&A S-3 of this notice, the amount in clause (ii) of the preceding
sentence is not less than the aggregate amount of dividends determined and
declared for the plan year that are determined in that same manner.
(b) Section 430(c)(7)(E)(iii) provides an exception for intra-group dividends,
under which dividends paid by one member of a controlled group (as defined in
§ 412(d)(3)) to another member of that group are not taken into account.
(c) For purposes of determining the excess shareholder payment amount,
pursuant to § 430(c)(7)(F), the term “plan sponsor” includes all members of the
plan sponsor’s controlled group. However, in the case of a plan established or
maintained jointly by an employer and an employee organization, the term “plan
sponsor” includes all members of the employer’s controlled group.
Q S-2: How is the adjusted net income for the prior plan year determined for
purposes of calculating the excess shareholder payment amount?
A S-2: (a) The adjusted net income for the prior plan year for purposes of
calculating the excess shareholder payment amount is the adjusted net income
(within the meaning of 29 CFR 4043.31(e)(1)) of the plan sponsor for the fiscal
year ending with or during that prior plan year (the applicable fiscal year),

25

determined before any reduction by reason of interest, taxes, depreciation, or
amortization. Adjusted net income is defined in 29 CFR 4043.31(e)(1) as the net
income before after-tax gain or loss on any sale of assets, as determined in
accordance with generally accepted accounting principles and practices.
Accordingly, the adjusted net income for purposes of calculating the excess
shareholder payment amount is the net income as determined in accordance
with generally accepted accounting principles and practices, but determined
before any reduction by reason of interest, taxes, depreciation, or amortization
and disregarding any gain or loss on any sale of assets. The amount of adjusted
net income for this purpose is deemed to be no less than zero.
(b) If the length of the plan year for which the excess shareholder payment
amount is determined and the fiscal year used to determine the adjusted net
income described in Q&A S-2 are not the same, the adjusted net income is
multiplied by a fraction, the numerator of which is the number of months in the
plan year for which the excess shareholder payment amount is determined and
the denominator of which is the number of months in the fiscal year for which the
adjusted net income is determined. For example, if a short plan year is 5 months
long, and the fiscal year is 12 months long, then the adjusted net income for the
prior plan year as determined in this Q&A S-2 is multiplied by 5/12 to determine
the adjusted net income used in calculating the excess shareholder payment
amount.
Q S-3: What does it mean for dividends to be determined and declared in the
same manner for at least 5 consecutive years?
A S-3: (a) The aggregate amount of dividends declared during the plan year for
which the installment acceleration amount is determined is deemed to be
determined in the same manner for at least 5 consecutive years if the dividends
are determined using the same formula (including the same specified dollar
amount, determined on either a per share basis, with appropriate adjustments for
stock splits and similar changes in capitalization, or on an aggregate basis) as
was used for dividends declared during the 60-month period immediately
preceding the first day of that plan year. Other examples of dividends
determined using the same formula include dividends that increase by a fixed
amount each year, dividends that increase by a fixed percentage each year, and
dividends that are a fixed percentage of income, earnings, or other consistently
applied measure of profitability.
(b) If, at any time during the 60 months immediately preceding the first day of the
plan year for which the installment acceleration amount is determined, a plan
sponsor omitted its periodic dividend or the sponsor has existed for fewer than 60
months prior to the first day of that plan year, the exception under
§ 430(c)(7)(E)(i)(II) does not apply.

26

Example S-1. (a) Assume that a plan sponsor that elected to use an alternative
amortization schedule for the shortfall amortization base established as of
January 1, 2011, has determined dividends in the same manner for the 60-month
period immediately preceding January 1, 2012. The plan sponsor continues to
determine dividends in the same manner for dividends declared as of
March 31, 2012, June 30, 2012, and December 31, 2012, but declares a reduced
dividend as of September 30, 2012. Assume the following dividends were
declared as a result:

Date dividends declared
March 31, 2012
June 30, 2012
September 30, 2012
December 31, 2012
Total dividends declared
during the plan year

Dividend
amount
$100,000
101,000
50,000
99,000
$350,000

Dividend determined in the same
manner as the 60-month period
ending December 31, 2011
Yes
Yes
No
Yes
N/A

(b) In accordance with Q&A S-1 of this notice, the excess shareholder payment
amount for the 2012 plan year reflects the total dividends declared during the
plan year, or $350,000. However, the amount of dividends eligible for the
exception under § 430(c)(7)(E)(i)(II) excludes the dividend declared
September 30, 2012, because that dividend was not determined in the same
manner as dividends declared during the 60-month period immediately preceding
the plan year for which the installment acceleration amount is calculated.
Accordingly, the amount of dividends taken into account for purposes of Q&A
S-1(a)(ii) of this notice is $300,000, and the excess shareholder payment
amount for the 2012 plan year is the excess of (i) the $350,000 in dividends
declared by the plan sponsor during the 2012 plan year plus the aggregate
amount paid for the redemption of stock during 2012, over (ii) the adjusted net
income for the plan sponsor for the fiscal year that ends during 2011 or $300,000
if larger.
(c) For any excess shareholder payment amount determined for the 2013 plan
year, no dividends are eligible for the exception under § 430(c)(7)(E)(i)(II),
because the dividends were not determined in the same manner throughout the
60-month period immediately preceding January 1, 2013.
Example S-2. (a) Assume that the sponsor of Plan A has an October 1 September 30 fiscal year and elects to use an alternative amortization schedule
for Plan A for its plan year beginning July 1, 2011. Assume further that the
dividends declared during the plan year beginning July 1, 2011, total $1,200,000,
payments made for the redemption of stock during the plan year beginning
July 1, 2011, total $500,000, and that the adjusted net income for the fiscal year
ending September 30, 2010, was $1,000,000.

27

(b) If the dividends declared during the plan year beginning July 1, 2011, were
determined in the same manner as those declared during the 60 months
preceding July 1, 2011 (that is, during the period beginning July 1, 2006, and
ending June 30, 2011), the exception under Q&A S-1 of this notice and
§ 430(c)(7)(E)(i)(II) applies to the dividends declared during the plan year
beginning July 1, 2011. In such a case, the excess shareholder payment amount
for the plan year beginning July 1, 2011, is equal to the excess of (i) the sum of
dividends declared by the plan sponsor during the plan year beginning
July 1, 2011, and amounts paid by the plan sponsor for the redemption of stock
during that plan year ($1,200,000 plus $500,000, or $1,700,000) over (ii) the
greater of the plan sponsor’s adjusted net income for the fiscal year beginning
October 1, 2009, and ending September 30, 2010, or dividends declared during
the plan year beginning July 1, 2011 (the greater of $1,000,000 or $1,200,000, or
$1,200,000). The excess shareholder payment amount for the plan year
beginning July 1, 2011, is therefore $500,000, which is the excess of $1,700,000
over $1,200,000.
(c) However, if none of the dividends declared during the plan year beginning
July 1, 2011, were determined in the same manner as those declared during the
60-month period beginning July 1, 2006, and ending June 30, 2011 (or if the
dividends declared during the 60-month period beginning July 1, 2006, and
ending June 30, 2011, were not determined in the same manner throughout that
period), then the exception under Q&A S-1 of this notice and § 430(c)(7)(E)(i)(II)
does not apply to the dividends declared during the plan year beginning
July 1, 2011. In that case, the excess shareholder payment amount for the plan
year beginning July 1, 2011, would be equal to $700,000, which is the excess of
(i) the sum of dividends declared by the plan sponsor during the plan year
beginning July 1, 2011, and amounts paid by the plan sponsor for the redemption
of stock redeemed during that plan year, or $1,700,000, over (ii) the plan
sponsor’s adjusted net income for the fiscal year beginning October 1, 2009, and
ending September 30, 2010 ($1,000,000).
Q S-4: How does the calculation of excess shareholder payment amounts
change if the installment acceleration amount is to be allocated among plans with
differing plan years?
A S-4: (a) If on any date during a calendar year (the applicable calendar year)
there are two or more plans within the controlled group for which an election has
been made to use an alternative amortization schedule (for which the restriction
period has not ended) and two or more of those plans have different plan years,
then, for all plan years that begin in the applicable calendar year, the excess
shareholder payment is determined as if the plan year were the calendar year.
Accordingly, subject to the special rules in Q&A S-5 through Q&A S-8 of this
notice, the excess shareholder payment amount for a plan year is calculated in
accordance with Q&A S-1 through Q&A S-3 of this notice, but (i) using dividends

28

declared by the plan sponsor and the aggregate amount paid for the redemption
of stock, as defined in Q&A S-1 of this notice, during the applicable calendar
year, (ii) using adjusted net income, as defined in Q&A S-2 of this notice, for the
fiscal year ending with or during the calendar year preceding the applicable
calendar year, and (iii) determining whether dividends have been determined and
declared in the same manner for at least 60 consecutive months in accordance
with Q&A S-3 of this notice by using the 60 months immediately preceding the
first day of the applicable calendar year as the relevant period. In such a case,
the installment acceleration amount is determined based on the applicable
calendar year in which the plan year begins because both the excess
shareholder payment amount and the excess compensation amount are
determined based on the applicable calendar year. 
 

(b) If the special rule of paragraph (a) applies for an applicable calendar year but
did not apply in the preceding calendar year, then, except as provided in
paragraph (c) of this Q&A S-4, the excess shareholder payment for any plan year
that began in the preceding calendar year and ends in the applicable calendar
year is determined as if the plan year were the preceding calendar year.
(c) The special rule in paragraph (b) of this Q&A S-4 does not apply if the date in
the applicable calendar year on which the conditions of paragraph (a) of this Q&A
S-4 are first satisfied is after the close of all the plan years that began in the prior
calendar year and end in the applicable calendar year. 
Example S-3. (a) The facts are the same as in Example S-2, except that the plan
sponsor also elects to use an alternative amortization schedule for Plan B, which
has a plan year beginning March 1. The shortfall amortization base for Plan B is
still in the restriction period for the plan year beginning March 1, 2011, and the
installment acceleration amount will be allocated between Plans A and B.
(b) The excess shareholder payment amount is determined based on the
calendar year in which the plan years begin. Accordingly, the excess
shareholder payment amount allocated to Plan A for the plan year beginning
July 1, 2011, and to Plan B for the plan year beginning March 1, 2011, is based
on (i) the dividends declared by the plan sponsor and amounts paid by the plan
sponsor for the redemption of stock as defined in Q&A S-1 of this notice during
the 2011 calendar year; and (ii) the plan sponsor’s adjusted net income as
defined in Q&A S-2 of this notice for the fiscal year beginning October 1, 2009,
and ending September 30, 2010. Dividends declared throughout the 2006
through 2010 calendar years (the 60-month period immediately preceding
January 1, 2011) must have been determined and declared in the same manner,
in order for any dividends to be eligible for the exception under
§ 430(c)(7)(E)(i)(II) and Q&A S-1 of this notice.
Q S-5: Section § 430(c)(7)(E)(ii) provides that only dividends declared, and
redemptions occurring, after February 28, 2010, are taken into account. How is
this rule applied?
29

A S-5: Only dividends declared, and redemptions occurring, after February 28,
2010, are taken into account in determining the excess shareholder payment
amount that is included in the installment acceleration amount for any plan year.
Accordingly, any dividends declared on or before February 28, 2010, are not
reflected in the installment acceleration amount without regard to when they are
paid. In contrast, any stock redemptions occurring after February 28, 2010, are
reflected in the installment acceleration amount without regard to when they were
announced.
Q S-6: Under what circumstances is a distribution of the stock of the employer
taken into account in determining the excess shareholder payment?
A S-6: Pursuant to § 305(a), a distribution by a corporation of its stock is
generally not treated as a dividend and is therefore not generally taken into
account in determining the excess shareholder payment. However, pursuant to
§ 305(b), a distribution of stock that is described in § 305(b)(1) through (5) (for
example, a distribution of stock that has the result of changing the proportionate
interests of the shareholders in the earnings and profits of the corporation) is
treated as a distribution of property to which § 301 applies and therefore is a
dividend that is taken into account in determining the excess shareholder
payment, provided that the distribution is paid out of earnings and profits as
described in § 316(a).
Q S-7: Under § 430(c)(7)(E)(iv), redemptions of stock that are made pursuant to
a plan maintained with respect to employees, or that are made on account of the
death, disability, or termination of employment of an employee or shareholder,
are not taken into account. How is this rule applied?
A S-7: (a) Redemptions of shares that are made pursuant to a plan maintained
with respect to employees (whether or not the plan is qualified) are not taken into
account when determining the amount of excess shareholder payment amounts.
For this purpose, a plan maintained with respect to employees that is not a
qualified plan does not include an agreement or arrangement that covers only a
single individual, but instead is limited to a plan that covers a category of
employees.
(b) Redemptions that are made on account of the death, disability, or termination
of employment of an employee or shareholder, are not taken into account in
determining the excess shareholder payment amount that is included in the
installment acceleration amount. For this purpose, a redemption is made on
account of death, disability, or termination of employment only if, as a result of
the death, disability, or termination of employment of the employee or
shareholder, either (i) the plan sponsor or any member of the plan sponsor’s
controlled group is required to redeem the stock held by the shareholder (even if
the shareholder is not required to tender the stock) or (ii) the shareholder is

30

required to tender the stock for redemption (even if the plan sponsor or a
member of the plan sponsor’s controlled group is not required to redeem the
stock tendered).
(c) For example, if a company’s bylaws require that the shares in the company
be held by active employees, an employee who terminates employment is
required to tender his/her shares in the company for redemption. Any amount
paid by the company to redeem these shares would not be taken into account for
purposes of determining an excess shareholder payment amount. However, if
the spouse of a deceased shareholder in a company that does not require shares
to be held by active employees voluntarily redeems the stock held by the
shareholder (under circumstances not otherwise described in clause (i) or (ii) of
paragraph (b) of this Q&A S-7), the amount paid by that company to redeem
these shares would be taken into account when determining an excess
shareholder payment amount.
Q S-8: Section 430(c)(7)(E)(v) provides an exception with respect to certain
preferred stock. How is this rule applied?
A S-8: Under this exception, dividends with respect to applicable preferred stock
are not taken into account in determining the excess shareholder payment
amount that is included in the installment acceleration amount to the extent that
dividends accrue with respect to that stock at a specified rate in all events and
without regard to the plan sponsor’s income, and interest accrues on any unpaid
dividends with respect to that stock. In addition, redemptions with respect to
applicable preferred stock are not taken into account in determining the excess
shareholder payment amount that is included in the installment acceleration
amount if dividends accrue with respect to that stock at a specified rate in all
events and without regard to the plan sponsor’s income, and interest accrues on
any unpaid dividends with respect to that stock. For this purpose, applicable
preferred stock is preferred stock that was originally issued before March 1,
2010, or that was originally issued prior to that date and is subsequently reissued
with otherwise identical terms. In addition, applicable preferred stock includes
preferred stock that was issued on or after March 1, 2010, and is held by an
employee benefit plan that is subject to the provisions of Title I of ERISA and that
is maintained for the benefit of employees and former employees of the plan
sponsor and their beneficiaries.
M. MERGERS AND ACQUISITIONS
Section 430(c)(2)(F)(iv) provides that the Secretary of the Treasury is to
prescribe rules for the application of installment acceleration amounts in any
case where there is a merger or acquisition involving a plan sponsor that has
elected to use an alternative amortization schedule in accordance with
§ 430(c)(2)(D).

31

Q M-1: How does a merger or acquisition affect the calculation of the excess
compensation amount described in section III.C and the excess shareholder
payment amount described in section III.S?
A M-1: (a) If a plan sponsor that elected to use an alternative amortization
schedule for any shortfall amortization base merges with, acquires, or is acquired
by another company that was not in its controlled group prior to the transaction,
and that other company either did not sponsor a single-employer defined benefit
plan or did not elect to use an alternative amortization schedule for any plan it
sponsored, any aggregate compensation amount with respect to employees of
such other company that would otherwise be taken into account prior to the date
of the merger or acquisition is disregarded for the purpose of calculating the
amounts in section III.C of this notice with respect to any installment acceleration
amounts determined for the plan sponsor. Similarly, any dividends declared by
such other company and any redemptions of stock of such other company that
occur prior to the date of the transaction are ignored for the purpose of
calculating the amounts in section III.S of this notice.
(b) If the other company involved in the transaction described in paragraph (a) of
this Q&A M-1 sponsors a plan for which an election was made to use an
alternative amortization schedule, the rule in paragraph (a) of this Q&A M-1 does
not apply. Accordingly, the installment acceleration amount is determined by
combining the compensation amounts and shareholder payment amounts for
both companies involved in the transaction as if they had been in the same
controlled group before and after the transaction. The installment acceleration
amount is then allocated as described in Q&A I-7 through Q&A I-10 of this notice.
(c) For purposes of this Q&A M-1, (i) any election to use an alternative
amortization schedule during the plan year in which the transaction occurs is
treated as an election that occurred prior to the date of the transaction and (ii) a
shortfall amortization base that is not in its restriction period for the plan year of
the transaction is disregarded when determining whether a plan sponsor has
elected to use an alternative amortization schedule.
(d) The rules of this Q&A M-1 apply regardless of whether the transaction is an
asset or a stock transaction.
Example M-1. (a) Assume that Company A and Company B are separate
companies that are not in the same controlled group, and that Company A
sponsors a single-employer defined benefit plan for which an election was made
to use an alternative amortization schedule for the 2010 plan year. Company B
sponsors a single-employer defined benefit plan but did not elect to use an
alternative amortization schedule. Company A acquires Company B effective
July 1, 2011. Consider the following employees:

32

Employee
Company
Compensation for 2011:
Paid before July 1, 2011
Paid on or after July 1, 2011
Total

Employee X
A
$400,000
1,100,000
$1,500,000

Employee Y
B
$750,000
750,000
$1,500,000

(b) The installment acceleration amount for 2011 for the plan sponsored by
Company A reflects the compensation paid to Employee X for all of 2011,
because Employee X was an employee of Company A prior to the date of the
acquisition. The threshold for determining excess compensation for 2011 is
$1,014,000. Therefore $486,000 of Employee X’s compensation is excess
compensation. However, only the compensation paid to Employee Y after
July 1, 2011, is considered when calculating the 2011 installment acceleration
amount for the plan sponsored by Company A, because Employee Y was not in
the same controlled group as Company A prior to the date of the acquisition.
Because the compensation paid to Employee Y on and after July 1, 2011, was
under $1,014,000, none of Employee Y’s compensation is considered excess
compensation.
Example M-2. (a) The facts are the same as in Example M-1, except that
Company B, not Company A, elected to use an alternative amortization schedule
for the 2010 plan year. The installment acceleration amount for 2011 for the plan
sponsored by Company B reflects the compensation paid to Employee Y for all of
2011, because Employee Y was an employee of Company B prior to the date of
the acquisition. Therefore, $486,000 of Employee Y’s compensation is excess
compensation for the purpose of determining the installment acceleration amount
for the 2011 plan year.
(b) Only the compensation paid to Employee X after July 1, 2011, is considered
when calculating the 2011 installment acceleration amount for the plan
sponsored by Company B, because Employee X was not in the same controlled
group as Company B prior to the date of the acquisition. As a result, $86,000 of
Employee X’s compensation is excess compensation for the purpose of
determining the installment acceleration amount for the 2011 plan year.
Q M-2: Are amounts paid to purchase stock pursuant to a merger or acquisition
reflected in the excess shareholder payment amount for the plan year in which
the transaction occurs?
A M-2: Any amounts paid to redeem or purchase stock pursuant to a merger or
acquisition are not reflected in the excess shareholder payment amount
determined according to section III.S of this notice, regardless of whether the
actual payment occurs during the plan year in which the transaction occurs.

33

E. ELECTIONS TO USE AN ALTERNATIVE AMORTIZATION SCHEDULE
Section 430(c)(2)(D)(iv)(III) provides that a plan sponsor’s election under
§ 430(c)(2)(D) is to be made at such time and in such form and manner as is
prescribed by the Secretary of the Treasury. Section 430(c)(2)(D)(iv)(III)
provides further that any such election may be revoked only with the consent of
the Secretary, after consultation with the Pension Benefit Guaranty Corporation
(PBGC).
Q E-1: How is an election made to use an alternative amortization schedule for a
plan year?
A E-1: An election made on or after January 1, 2011, must be made by the plan
sponsor, by providing written notification of such election to both the plan’s
enrolled actuary and the plan administrator. Such election must be signed and
dated by the plan sponsor and must include all of the following information:
(1) The name of the plan;
(2) The plan number;
(3) The name of the plan sponsor;
(4) The plan sponsor’s mailing address;
(5) The plan sponsor’s employer identification number;
(6) Which of the two alternative amortization schedules is being elected;
(7) The plan year for which the election is being made;
(8) Whether an alternative amortization schedule has been elected for
another year, and, if so, a statement that the same alternative amortization
schedule is being elected; and
(9) A statement that the plan sponsor will notify the PBGC and plan
participants and beneficiaries pursuant to § 430(c)(2)(D)(vi) of the Code
and ERISA section 303(c)(2)(D)(vi).
Q E-2: What is the deadline for making the election?
A E-2: The election must be made by the latest of: (i) the last day of the plan
year for which the election is made, (ii) 30 days after the valuation date for the
plan year for which the election is made, or (iii) January 31, 2011. For example,
if the valuation date for a plan is the first day of the plan year, an election for the
plan year that begins on January 1, 2009, or January 1, 2010, must be made by
January 31, 2011; for the plan year that begins on January 1, 2011, the election
must be made by December 31, 2011.
Q E-3: How does the ability to elect an alternative amortization schedule apply to
multiple employer plans?
A E-3: (a) In the case of a multiple employer plan to which § 413(c)(4)(A)
applies, the rules of § 430 and this section are applied separately for each

34

employer under the plan as if each employer maintained a separate plan. Thus,
each employer under a multiple employer plan may elect to use an alternative
amortization schedule independent of the elections of other employers under the
plan.
(b) In the case of a multiple employer plan to which § 413(c)(4)(A) does not
apply, the rules of § 430 and this notice are applied as if all participants in the
plan were employed by a single employer, and any reference to the plan sponsor
means the plan administrator within the meaning of § 414(g).
N. NOTIFICATION TO PARTICIPANTS, BENEFICIARIES, AND THE PBGC
Section 303(c)(2)(D)(vi)(I) of ERISA and § 430(c)(2)(D)(vi)(I) of the Code require
a plan sponsor that elects funding relief to give notice of the election to
participants and beneficiaries of the plan (pension funding relief notice).
Section 303(c)(2)(D)(vi)(II) of ERISA and § 430(c)(2)(D)(vi)(II) of the Code
require the plan sponsor to inform the PBGC of such election in such form and
manner as the Director of the PBGC may prescribe. The PBGC has informed the
Treasury Department and the Service that the form and manner for complying
with this requirement are the rules described in Q&A N-6 and Q&A N-7 of this
section III.N.
Q N-1: When must the pension funding relief notice be provided to participants
and beneficiaries?
A N-1: The pension funding relief notice must be provided to participants and
beneficiaries of the plan by 120 days after the end of the plan year for which an
alternative amortization schedule is elected, or by May 2, 2011, if later. For
example, if an alternative amortization schedule is elected for a plan year
beginning June 1, 2010, then the notice must be provided to participants and
beneficiaries by September 28, 2011. If the election for a plan is made
simultaneously for two plan years, the notices for both elections can be combined
as long as the notice identifies both years for which the election is made.
Q N-2: Which participants and beneficiaries must receive the notice?
A N-2: Except as otherwise provided in this section III.N, a pension funding relief
notice is required to be provided to all plan participants and beneficiaries.
However, the pension funding relief notice does not have to be provided to any
person who either became a plan participant or beneficiary after the last day of
the last plan year ending before the notice is due or ceased to be a participant or
beneficiary prior to the date on which the pension funding relief notice is
provided.
Q N-3: How does the notice requirement affect multiple employer plans?

35

A N-3: (a) In the case of a multiple employer plan to which § 413(c)(4)(A)
applies, the rules of § 430 are applied separately for each employer under the
plan. Accordingly, the notice in this section III.N must be provided only to those
participants or beneficiaries as described in Q&A N-2 of this notice who are
associated with a plan sponsor which elects to use an alternative amortization
schedule.
(b) In the case of a multiple employer plan to which § 413(c)(4)(A) does not
apply, the rules of § 430 are applied as if all participants in the plan were
employed by a single employer. Therefore, if an election is made to use an
alternative amortization schedule, such election applies to the entire plan and the
notice must be provided to all participants and beneficiaries of the plan as
described in Q&A N-2 of this notice.
Q N-4: What information is required to be in a pension funding relief notice?
A N-4: (a) A pension funding relief notice must provide (1) the name of the plan
for which the election has been made, (2) the plan year for which the election has
been made, (3) a general description of the effect of the election, including the
fact that the election will delay pension funding and which of the two schedules
has been elected, and (4) the name, address, and telephone number of the plan
administrator or other contact person from whom more information may be
obtained.
(b) A pension funding relief notice must be written in a manner calculated to be
understood by the average plan participant or beneficiary. In addition, the notice
must be written in such a manner that the average participant or beneficiary will
understand the significance of the required information in the notice. While a
pension funding relief notice may include any additional information that is
necessary or helpful for recipients to understand the required information in the
notice, the notice should not have the effect of misleading or misinforming
recipients or of distracting recipients from the required information in the notice.
A pension funding relief notice must be a separate notice and cannot be
combined with other information. However, a pension funding relief notice can
be provided at the same time as another notice is provided; for example, a
pension funding relief notice does not fail to meet the requirements of this Q&A
N-4 merely because it is provided at the same time as a notice under section
101(f) of ERISA.
(c) The following examples illustrate information that satisfies the requirements of
paragraph (a) of this Q&A N-4:
(i) ALTERNATIVE 1 -- pension funding relief notice reflecting an election for a
plan for one plan year

36

Notice Regarding [ENTER NAME OF PLAN]
The employer sponsoring your pension plan has made an election permitted
under Federal law to delay funding for the plan. The election applies to the plan
year beginning on [ENTER DATE] and ending on [ENTER DATE].
[ALTERNATIVE IF 2 PLUS 7-YEAR SCHEDULE IS ELECTED: Without the
election, Federal law generally requires that any increase in the amount by which
the plan is underfunded for a plan year be paid off over 7 years. However, the
election allows the increase in the amount by which the plan is underfunded for
this plan year to be paid off over 9 years, with the payments for the first 2 years
limited to interest on that increase.]
[ALTERNATIVE IF 15-YEAR SCHEDULE IS ELECTED: Without the election,
Federal law generally requires that any increase in the amount by which the plan
is underfunded for a plan year be paid off over 7 years. However, the election
allows the increase in the amount by which the plan is underfunded for this plan
year to be paid off in smaller annual payments over 15 years.]
If you have any questions, contact [ENTER NAME, ADDRESS, AND
TELEPHONE NUMBER FOR CONTACT INFORMATION].
(ii) ALTERNATIVE 2 -- pension funding relief notice reflecting an election for a
plan for two plan years
Notice Regarding [ENTER NAME OF PLAN]
The employer sponsoring your pension plan has made elections permitted under
Federal law to delay funding for the plan. The elections apply to the plan year
beginning on [ENTER DATE] and ending on [ENTER DATE], and to the plan
year beginning on [ENTER DATE] and ending on [ENTER DATE].
[ALTERNATIVE IF 2 PLUS 7-YEAR SCHEDULE IS ELECTED: Without the
elections, Federal law generally requires that any increase in the amount by
which the plan is underfunded for a plan year be paid off over 7 years. However,
for each of these years, the election allows the increase in the amount by which
the plan is underfunded for the plan year to be paid off over 9 years, with the
payments for the first 2 years limited to interest on that increase.]
[ALTERNATIVE IF 15-YEAR SCHEDULE IS ELECTED: Without the elections,
Federal law generally requires that any increase in the amount by which the plan
is underfunded for a plan year be paid off over a period of 7 years. However, the
election allows the increase in the amount by which the plan is underfunded for
each of these plan years to be paid off in smaller annual payments over 15
years.

37

If you have any questions, contact [ENTER NAME, ADDRESS, AND
TELEPHONE NUMBER FOR CONTACT INFORMATION].
Q N-5: What are the acceptable methods of providing the pension funding relief
notice? In particular, can the notice be provided electronically?
A N-5: The pension funding relief notice must be in writing and may be furnished
in any paper or electronic form to the extent such form is reasonably accessible
to persons to whom the notice is required to be provided. Permissible electronic
methods include those permitted under regulations of the Department of Labor at
29 C.F.R. § 2520.104b-1(c) and those described at § 54.4980F-1, Q&A-13(c).
Q N-6: How does a plan sponsor electing pension funding relief comply with the
requirement to notify the PBGC of such election?
A N-6: A copy of an election made for a plan that is covered by the PBGC must
be e-mailed to the PBGC at [email protected].
The subject line of the e-mail must contain the plan sponsor’s employer
identification number, the plan number, and the name of the plan. See Q&A T-1
of this notice for additional information that may be required for elections made
before January 1, 2011.
Q N-7: What is the deadline for notifying the PBGC?
A N-7: PBGC notification must be made by the later of: (i) 30 days after the date
the election is made or (ii) January 31, 2011.
CP. ELIGIBLE CHARITY PLANS
Section 202(b) of PRA 2010 provides that delayed effective dates under section
104 of PPA ’06 are applicable to eligible charity plans.
Q CP-1: What is an eligible charity plan for purposes of section 202(b) of PRA
2010?
A CP-1: (a) A plan is an eligible charity plan for a plan year if it is maintained by
more than one employer, each of which is described in § 501(c)(3), determined
without regard to whether the employers are members of the same controlled
group.
(b) In accordance with section 104(a) of PPA ’06 as amended by section 202(b)
of PRA 2010, the delayed effective dates under section 104 with respect to an
eligible charity plan only apply to a plan that was in existence as of July 26, 2005,
and that was an eligible charity plan for the plan year that includes that date.
Under section 104 of PPA ’06, the rules of §§ 430 and 436 do not apply with
respect to an eligible charity plan until the earlier of (i) the first plan year in which

38

the plan ceases to be an eligible charity plan and (ii) the first plan year beginning
on or after January 1, 2017. Therefore, the delay in effective dates under section
104 with respect to an eligible charity plan does not apply to a plan established
after July 26, 2005, or to a plan that was not an eligible charity plan on that date.
In addition, for plan years that begin before January 1, 2017, §§ 430 and 436
apply to a plan that was an eligible charity plan on July 26, 2005, beginning with
the first plan year during which the plan fails to be an eligible charity plan.

R. REPORTING REQUIREMENTS
Notice 2010-55 (2010-33 I.R.B. 253) provides that, in the case of a plan year that
ends before guidance on the special funding rules under PRA 2010 is issued, a
plan sponsor will be permitted to elect to use an alternative amortization
schedule under PRA 2010 without regard to whether the Form 5500 (and
Schedule SB) has been filed for that plan year. However, Notice 2010-55 did not
preclude plan sponsors from electing to use the special funding rules before such
guidance was issued, nor did it preclude enrolled actuaries from reporting
shortfall amortization installment amounts on the Schedule SB reflecting their
understanding of the calculations under the alternative amortization schedules.
The Service expects that some plan sponsors may have made an election to use
an alternative amortization schedule prior to the issuance of this notice, and that
at least some of these elections may not have complied with the requirements of
this notice.
In addition, the Service expects that some plan sponsors have elected (or will
elect) to use an alternative amortization schedule for a plan year that ended prior
to the issuance of this notice. For some of these plans, the Schedule SB for that
plan year may reflect (1) a minimum required contribution that does not take the
alternative amortization schedule into account, or (2) a minimum required
contribution that takes the alternative amortization schedule into account but is
different than the amount determined using the rules in this notice.
Q R-1: If, in accordance with Notice 2010-55, the plan sponsor elected to use an
alternative amortization schedule for the 2008 or 2009 plan year, and the election
was not reflected in the computations on the Schedule SB filed with the Form
5500 or Form 5500-SF for the relevant plan year, how should the reduced
shortfall amortization installments be reflected in the reporting for the plan?
A R-1: (a) The filing of an amended Form 5500 or Form 5500-SF for the 2008 or
2009 plan year is not required solely because the plan sponsor elected to use an
alternative amortization schedule for that plan year which was not reflected in the
computations on the Schedule SB. However, the Schedule SB filed with the
Form 5500 or Form 5500-SF for a subsequent plan year no later than the 2010
plan year must accurately reflect the effect of any election to use an alternative

39

amortization schedule for the 2008 or 2009 plan year on the calculation of the
minimum required contribution, determined in accordance with this notice. To
the extent that the amounts shown on the Schedule SB for the subsequent plan
year are different than the amounts shown on the Schedule SB for prior years,
this difference should be explained in attachments to the Schedule SB for such
subsequent plan year as explained in paragraphs (b), (c), (d), and (e) of this
Q&A R-1 (whichever apply).
(b) A plan sponsor’s election to use an alternative amortization schedule for the
2008 plan year will affect the minimum required contribution and either the
amount of excess contributions or the amount of the unpaid minimum required
contribution for the 2008 and 2009 plan years. If, in accordance with Q&A T-2 of
this notice, the plan sponsor elects to add an additional amount to the plan’s
prefunding balance as a result of the election for the 2008 plan year made after
the Schedule SB for the 2009 plan year is filed, the plan’s prefunding balance as
of the beginning of the 2009 plan year as reported on Line 13 of the Schedule SB
filed for the 2009 plan year will be different than the actual amount of the
prefunding balance as of the beginning of the 2009 plan year that must be
reported on Line 7 of the Schedule SB filed for the 2010 plan year. In this
situation, this difference should be explained in an attachment to Line 7 of the
Schedule SB filed for the 2010 plan year. Additionally, the election for the 2008
plan year will affect the amount of excess contributions for the 2009 plan year.
Accordingly, the amount reported on Line 11a of the Schedule SB for the 2010
plan year will be different than the amount reported on Line 38 of the Schedule
SB filed for the 2009 plan year. The attachment already described in the
instructions for Line 9 of the Schedule SB filed for the 2010 plan year is an
appropriate means for providing an explanation of this difference.
(c) A plan sponsor’s election to use an alternative amortization schedule for the
2008 plan year will affect the plan’s minimum required contribution for both the
2008 and 2009 plan years. If an unpaid minimum required contribution was
reported on Line 40 of the Schedule SB filed for the 2009 plan year before the
election to use an alternative amortization schedule was made, then the amount
on Line 28 of the Schedule SB for the 2010 plan year will be different than that
amount. The attachment already described in the instructions for Line 9 of the
Schedule SB filed for the 2010 plan year is an appropriate means for providing
an explanation of this difference.
(d) If the plan sponsor’s election to use an alternative amortization schedule for
the 2008 plan year is first reflected on the Schedule SB for the 2009 plan year,
the amount on Line 11a of the Schedule SB for the 2009 plan year will be
different than Line 38 of the Schedule SB for the 2008 plan year. The
attachment already described in the instructions for Line 9 of the Schedule SB for
the 2009 plan year is an appropriate means for providing an explanation of this
difference.

40

(e) A plan sponsor’s election to use an alternative amortization schedule for the
2009 plan year after the Schedule SB is filed for that plan year will affect the
minimum required contribution and either the amount of excess contributions or
the amount of the unpaid minimum required contribution for the 2009 plan year.
Accordingly, the amount reported on Line 11a of the Schedule SB for the 2010
plan year will be different than the amount reported on Line 38 of the Schedule
SB filed for the 2009 plan year (or in the case of a change in the amount of the
unpaid minimum required contribution the amount reported on Line 28 of the
Schedule SB for the 2010 plan year will be different than the amount reported on
Line 40 of the Schedule SB for the 2009 plan year). The attachment already
described in the instructions for Line 9 of the Schedule SB filed for the 2010 plan
year is an appropriate means for providing an explanation of this difference.
Q R-2: If the plan sponsor elected to use an alternative amortization schedule for
the 2008 or 2009 plan year before the Form 5500 or Form 5500-SF for those
plan years were filed, and the calculations of the plan’s alternative amortization
schedule reflected on the forms filed before this guidance is reflected are
different than the calculations required under this notice, how should the revised
calculations be reflected in the reporting for the plan?
A R-2: (a) The minimum required contribution for the affected plan year must be
calculated in accordance with the rules of this notice, regardless of whether the
minimum required contribution amount was originally determined using a
reasonable interpretation of the statute.
(b) The filing of an amended Form 5500 or Form 5500-SF for the 2008 or 2009
plan year is not required solely to reflect changes in the calculation of the
minimum required contribution as a result of applying the rules in this notice.
However, the Schedule SB filed with the Form 5500 or Form 5500-SF for a
subsequent plan year no later than the 2010 plan year must accurately reflect the
effect of any election to use an alternative amortization schedule for the 2008 or
2009 plan year on the calculation of the minimum required contribution
determined in accordance with this notice. To the extent that the amounts shown
on the Schedule SB for the subsequent plan year are different than the amounts
shown on the Schedule SB for prior years, this difference should be explained in
attachments to the Schedule SB for the 2010 plan year as explained in Q&A R-1.
Q R-3: May a plan sponsor’s election to use an alternative amortization schedule
for the 2008 or 2009 plan year be reflected in the filing of an amended Form
5500 or Form 5500-SF for the 2008 or 2009 plan year (or both, as applicable)?
A R-3: Yes. In lieu of the reporting procedure described in Q&A R-1 and Q&A
R-2 of this notice, an amended Form 5500 or Form 5500-SF with a revised
Schedule SB showing the amounts based on the alternative amortization
schedule and the provisions of this notice is permitted to be filed.

41

Q R-4: How should the plan sponsor respond to any inquiries from the Service
regarding Form 5330 if the plan sponsor elected to use an alternative
amortization schedule after the Form 5500 or Form 5500-SF was filed with a
Schedule SB that did not reflect the reduced shortfall amortization installments?
A R-4: (a) If a plan sponsor expects that an unpaid minimum required
contribution shown on the Schedule SB will be eliminated by an election that the
plan sponsor intends to make to use an alternative amortization schedule, Form
5330 should not be filed. However, when a Schedule SB showing an unpaid
minimum required contribution is filed, and the plan sponsor does not timely file a
Form 5330 to pay the associated excise tax under § 4971(a), the Service will
normally send a notice that informs the plan sponsor that the Form 5330 and the
excise tax are due. In this case, the plan sponsor should respond to the notice,
advising the Service that the reported unpaid minimum required contribution will
be eliminated by an election to use an alternative amortization schedule and
providing supporting evidence thereof.
(b) If the plan sponsor expects to have an unpaid minimum required contribution
for the plan year once the alternative amortization schedule is reflected but did
not file a Form 5330 when due, the plan sponsor should file a Form 5330
reflecting the corrected unpaid minimum required contribution and pay the excise
tax under § 4971(a) as soon as possible in order to minimize interest and penalty
charges.
Q R-5: How should the effect of the plan sponsor’s election to use an alternative
amortization schedule be reflected on the Schedule SB?
A R-5: (a) If a plan sponsor’s election to use an alternative amortization
schedule is reflected on the Schedule SB for a plan year, the shortfall
amortization installment reported on Line 32a of the Schedule SB (and the
information reported in the attachment to Line 32a) must reflect the calculation of
the installment as determined under this notice. However, any Schedule SB filed
for plan years ending before December 17, 2010, is not required to reflect the
election. See Q&A R-1 and Q&A R-2 of this notice for rules regarding
reconciliation of amounts affected by the election when filing Schedule SB for the
plan for a subsequent year.
Q R-6: How are the rules of this section III.R applied to plans for which Schedule
SB is not required to be filed, pursuant to the instructions for Form 5500-EZ and
Form 5500-SF?
A R-6: Schedule SB is not required to be filed for plans for which Form 5500-EZ
is filed and certain plans for which Form 5500-SF is filed. For these plans, the
Schedule SB must be completed (including being signed by the enrolled actuary)
and delivered to the plan administrator, who must retain it. With respect to these

42

plans, the rules of this section III.R are applied by substituting the completion and
delivery of the Schedule SB for the filing of the Schedule SB.
T. TRANSITION RULES
Q T-1: What are the consequences if a plan sponsor made an election to use an
alternative amortization schedule prior to January 1, 2011, but the election did
not include all of the information required in Q&A E-1 of this notice?
A T-1: (a) If a plan sponsor made an election to use an alternative amortization
schedule prior to January 1, 2011, but the sponsor did not fulfill all the
requirements pertaining to an election to use an alternative amortization
schedule in Q&A E-1 of this notice, the fact that the sponsor did not meet all such
requirements does not invalidate the election, and does not permit the sponsor to
revoke that election without receiving approval from the IRS.
(b) Any sponsor that made such an election must notify participants of that
election by the deadline set forth in Q&A N-1 of this notice regardless of whether
the election to use an alternative amortization schedule was made before, on, or
after January 1, 2011.
(c) Any sponsor that made such an election for a plan that is covered by the
PBGC must include in the email notification to the PBGC described in Q&A N-6
of this notice any information described in Q&A E-1 of this notice whether or not
that information was included in the election.
Q T-2: If a plan sponsor’s election to use an alternative amortization schedule for
a plan year creates or increases the excess contributions for that plan year after
the deadline for making an election to increase the prefunding balance for that
year, can the plan sponsor still elect to increase the prefunding balance by the
additional excess contributions?
A T-2: (a) The minimum required contribution for a plan may decrease as a
result of an election to use an alternative amortization schedule when the
provisions of this notice are applied. This can occur either if the plan sponsor
makes an election to use an alternative amortization schedule that was not
reflected in the Schedule SB originally prepared for the plan year, or if the plan’s
enrolled actuary had prepared the Schedule SB reflecting an election to use an
alternative amortization schedule and showing a minimum required contribution
amount that was larger than the amount required under this notice.
(b) If a plan sponsor timely made a valid standing election in accordance with
§ 1.430(f)-1(f)(1)(ii) to add the maximum amount to the prefunding balance, then
any adjustments made to the minimum required contribution in accordance with
this notice will automatically increase the prefunding balance if the minimum
required contribution is reduced. However, a plan sponsor can make a written

43

election to temporarily suspend the standing election retroactively so that all or
part of the excess contributions created as a result of applying the provisions of
this notice are not added to the prefunding balance.
(c) If a plan sponsor had not made a standing election for the affected plan year
(or had temporarily suspended a standing election retroactively, as described in
paragraph (b) of this Q&A T-2), the plan sponsor may make an election to
increase the prefunding balance by an amount no greater than the amount of the
increase in excess contributions for the plan year resulting from the election to
use an alternative amortization schedule when the provisions of this notice are
applied.
(d) Plan sponsors that wish to increase the prefunding balance by any increase
in excess contributions that results from an election to use an alternative
amortization schedule in accordance with the provisions of this notice should
take into account the effect of the increase in the prefunding balance on the
adjusted funding target attainment percentage (“AFTAP”) as defined in
§ 1.436-1(j)(1), and the implications of that change on compliance with the
requirements of section 206(g) of ERISA and § 436 of the Code for plan years
beginning with the year to which that increase applies.
Q T-3: If a plan sponsor elected to use the funding standard carryover balance
or the prefunding balance (funding balances) to offset the minimum required
contribution for a plan year, and the minimum required contribution is decreased
as a result of making an election to apply an alternative amortization schedule in
accordance with the provisions of this notice, can the election to use the funding
balance(s) be revoked?
A T-3: (a) The minimum required contribution for a plan may decrease when a
plan sponsor makes an election to use an alternative amortization schedule in
accordance with the provisions of this notice. This can occur either if the plan
sponsor makes an election to use an alternative amortization schedule that was
not reflected in the Schedule SB originally prepared for the plan year, or if the
plan’s enrolled actuary had prepared the Schedule SB reflecting an election to
use an alternative amortization schedule and showing a minimum required
contribution that was larger than the amount required under this notice.
(b) If the plan sponsor had made a timely, valid standing election under
§ 1.430(f)-1(f)(1)(ii) to use the funding balances to offset the minimum required
contribution, the amount of the funding balances used to offset the minimum
required contribution will automatically be adjusted to reflect the amount of the
revised minimum required contribution. However, a plan sponsor can make a
written election to temporarily suspend the standing election retroactively so that
the amount of the funding standard carryover balance and the prefunding
balance is unchanged as a result of applying the provisions of this notice.

44

(c) Section 1.430(f)-1(f)(3) provides in general that elections with respect to the
plan’s prefunding balance or funding standard carryover balance are irrevocable
and must be unconditional. However, § 1.430(f)-1(f)(3)(ii) provides that an
election to use the prefunding balance or funding standard carryover balance to
offset the minimum required contribution for a plan year is permitted to be
revoked to the extent the amount the plan sponsor elected to use to offset the
minimum required contribution exceeds the minimum required contribution for the
plan year. This election must generally be made by the end of the plan year for
which the election was made (if the valuation date is the first day of the plan
year) or the due date (including extensions) for filing Schedule SB of Form 5500
(if the valuation date is not the first day of the plan year). However, a plan
sponsor is permitted to make an additional election under § 1.430(f)-1(f)(3)(ii), to
the extent that making an election to use an alternative amortization schedule in
accordance with the provisions of this notice reduced the minimum required
contribution for the plan year.
(d) Except as provided in paragraph (c) of this Q&A T-3, a plan sponsor is not
permitted to revoke an affirmative election to use the funding balances to offset
the minimum required contribution regardless of whether it was made before or
after the application of the provisions of this notice.
(e) Plan sponsors who wish to adjust the funding standard carryover balance or
the prefunding balance under paragraphs (b) or (c) above should take into
account the effect of that adjustment on the AFTAP as defined in § 1.436-1(j)(1),
and the implications of that change on compliance with the requirements of
section 206(g) of ERISA and § 436 of the Code for plan years beginning with the
year to which that adjustment applies.
Q T-4: What is the deadline for making the elections described in Q&A T-2 and
Q&A T-3 of this notice?
A T-4: Any of the elections described in Q&A T-2 or Q&A T-3 of this notice
(relating to changes in elections with respect to the funding standard carryover
balance and the prefunding balance) must be made no later than the due date
that would otherwise apply for making the election under § 1.430(f)-1(f) or
March 31, 2011, if later.
IV. PAPERWORK REDUCTION ACT
The collections of information contained in this notice have been reviewed and
approved by the Office of Management and Budget in accordance with the
Paperwork Reduction Act (44 U.S.C. § 3507) under control number 1545-2196.
An agency may not conduct or sponsor, and a person is not required to respond
to, a collection of information unless the collection of information displays a valid
OMB control number.

45

The collections of information in this notice are in section III of this notice. The
collections of information are required to determine the application of the special
funding rules under § 430(c)(2)(D) and to comply with the statutory notice
requirements related to those rules. The collections of information are
mandatory for those plan sponsors making an election to apply the special
funding rules. The likely respondents are sponsors of single employer defined
benefit plans.
For all information except for Q&A N-6, Q&A N-7, and Q&A T-1(c) of this notice
(relating to information provided to the Pension Benefit Guaranty Corporation
(PBGC)), the estimated total number of respondents is 34,100 plans. The
estimated annual burden per respondent varies from 30 minutes to 50 minutes,
depending on individual circumstances, with an estimated average of 45 minutes.
The estimated total annual reporting and/or recordkeeping burden is 25,700
hours.
The information collected in Q&A N-6, Q&A N-7, and Q&A T-1(c) applies only to
single-employer defined benefit plans covered by the PBGC. For this
information, the estimated total number of respondents is 13,820 plans. The
estimated annual burden per respondent/recordkeeper is 15 minutes.1 The
estimated total annual reporting and/or recordkeeping burden is 3,455 hours.
Estimates of the annualized cost to respondents for the hour burdens shown are
not available at this time.
Books or records relating to a collection of information must be retained as long
as their contents may become material in the administration of any internal
revenue law. Generally, tax returns and tax return information are confidential,
as required by § 6103.
V. DRAFTING INFORMATION
The principal author of this notice is Carolyn Zimmerman of the Employee Plans,
Tax Exempt and Government Entities Division. For further information regarding
this notice, please contact the Employee Plans taxpayer assistance answering
service at 1-877-829-5500 (a toll free number) or e-mail Ms. Zimmerman at
[email protected].

1

The PBGC expects that single-employer plans will incur this burden twice, in 2011 and 2012.
The figures shown represent the average annual number of respondents and reporting and/or
recordkeeping burden over a 3-year period.

46


File Typeapplication/pdf
File TitleMicrosoft Word - n-11-03.doc
AuthorInternal Revenue Service
File Modified2010-12-17
File Created2010-12-17

© 2024 OMB.report | Privacy Policy